Универсальный внешний накопитель для всех iOS-устройств, совместим с PC/Mac, Android
Header Banner
8 800 100 5771 | +7 495 540 4266
c 9:00 до 24:00 пн-пт | c 10:00 до 18:00 сб
0 Comments

Содержание

9.2 Электрическое напряжение – fizikalexcras

Интерактивное изложение материала по теме

Электрическое напряжение. Измерение напряжения  Подборка заданий “Электрическое напряжение, сила электрического тока”

Слайд-шоу “Солнечные батареи”
Слайд-шоу “Электрическое напряжение в природе”
Рисунок “Вольтметр” 
Подборка заданий “Электрическое напряжение, сила электрического тока”
Слайд-шоу “Опасное напряжение”
Слайд-шоу “Подключение приборов к электрической сети”

Напряжение электрического тока

 Электрический ток – это проходящие через проводник электроны, несущие отрицательный заряд. Объем этого заряда или, иными словами, количество электричества характеризует силу тока. Сила тока одинакова на всех участках цепи. Электроны не могут исчезать или «спрыгивать» с проводов. Поэтому, силу тока мы можем измерить в любом месте электрической цепи. Однако, будет ли одинаковым действие тока на разные участки этой цепи? Проходя по проводам, ток лишь слегка их нагревает, не совершая при этом большой работы.

Проходя же через спираль электрической лампочки, ток не просто сильно нагревает ее, он нагревает ее до такой степени, что она, раскаляясь, начинает светиться. То есть в данном случае ток совершает большую работу.

Определение электрического напряжения

Определение: напряжение электрического тока – это величина, показывающая, какую работу совершило поле при перемещении заряда от одной точки до другой. Напряжение в разных участках цепи будет различным. Напряжение на участке пустого провода будет совсем небольшим, а напряжение на участке с какой-либо нагрузкой будет гораздо большим, и зависеть величина напряжения будет от величины работы, произведенной током.

Напряжение  – это физическая величина, характеризующая действие электрического поля на заряженные частицы.

Напряжение показывает, какую работу совершает электрическое поле по перемещению единицы заряда на данном участке цепи. 

Постоянный ток: Напряжение.

Измеряют напряжение в вольтах (1 В). Для определения напряжения существует формула:
где U – напряжение,
A – работа, совершенная током по перемещению заряда q  на некий участок цепи.  Электрическое напряжение

Напряжение на полюсах источника тока


Напряжение на полюсах источника тока означает потенциальную величину энергии, которую может источник придать току. Это как давление воды в трубах. Эта величина энергии,

которая будет израсходована, если к источнику подключить некую нагрузку. Поэтому, чем большее напряжение у источника тока, тем большую работу может совершить ток.

Вольтметр

Для измерения напряжения существует прибор, называемый вольтметром. В отличие от амперметра, он подключается  параллельно нагрузке. В таком случае вольтметр показывает величину напряжения, приложенного к нагрузке. Для измерения напряжения на полюсах источника тока, вольтметр подключают непосредственно к полюсам прибора.

Что такое Электрическое напряжение – Определение, измерение

Большинство людей в быту могут оперировать таким понятием как электрическое напряжение. Практически все знают, что бытовая розетка находится под напряжением 220В, а пальчиковая батарейка выдает напряжение всего в 1.5В. При этом далеко не каждый человек, окончивший среднюю школу или даже технический ВУЗ в состоянии ответить, что же все-таки означает термин электрическое напряжение. В этом материале мы постараемся ответить на этот вопрос, по возможности не прибегая к сложной математике.

Определение электрического напряжения

В учебниках по физике и электротехнике можно встретить разные определения электрического напряжения. Одно из них звучит следующим образом: электрическое напряжение между двумя точками пространства равно разности потенциалов электрического поля в этих точках. Математически это записывается так:

U=φ_a-φ_b (1).

Где U – электрическое напряжение, а φ_a и φ_b потенциалы электрического поля в точках A и B соответственно.

Если мы не знаем что такое потенциал электрического поля в точке, то приведенное выше определение мало проясняет вопрос, что же такое электрическое напряжение. Под потенциалом электрического поля в точке понимают работу, по перемещению единичного заряда совершаемую электрическим полем из данной точки в точку с нулевым потенциалом. На первый взгляд определение электрического потенциала кажется довольно сложным. Например, не совсем понятно, где находится точка с нулевым потенциалом.

 Для начала нужно запомнить, что электрический потенциал это работа по переносу единичного заряда. Если обратиться к формуле (1) то станет ясно, что электрическое напряжение не что иное, как разность двух работ. То есть электрическое напряжение, тоже есть работа. Отсюда мы приходим ко второму определению. Электрическое напряжение численно равно работе по переносу единичного электрического заряда из точки А в точку В. При этом φ_a и φ_b это потенциальная энергия которой обладает единичный заряд в точках А и В соответственно.

Для лучшего понимания изложенного выше можно привести следующую аналогию. Любое тело, находящееся на некотором расстоянии от Земли обладает потенциальной энергией. Для того чтобы поднять тело выше придется выполнить некоторую работу. Величина этой работы будет равна разности потенциальных энергий, которыми обладает тело на разной высоте. Похожую картину мы наблюдаем, когда мы имеем дело с электрическим полем.

Что касается точки пространства, в которой электрический заряд обладает нулевым электрическим потенциалом, то в теории электричества эту точку можно выбрать произвольно. Связанно это с тем, что электрическое поле «потенциально». Чтобы прояснить этот термин придется прибегнуть к высшей математике, а мы решили этого избежать. На практике специалисты в области электротехники в качестве точек с нулевым потенциалом часто выбирают поверхность Земли. И многие измерения выполняют относительно нее.

Электрические поля могут быть постоянными (неизменными во времени) и переменными. Переменные электрические поля могут изменяться по различным математическим законам. В технике чаще всего используются переменные электрические поля, которые изменяются по закону синуса.

В случае переменного электрического поля мгновенное значение разности потенциалов между двумя точками можно вычислить по следующей формуле:

u(t)=U_m  sin⁡〖(ωt)〗 (2).

Здесь u – мгновенное значение напряжения; Um – максимальное значение напряжения; ω – частота, t – время.

Измерение электрического напряжения


Электрическое напряжение измеряют с помощью вольтметров. Для измерения напряжения (разности потенциалов) на участке электрической цепи щупы вольтметра подключают к концам этого участка и по шкале считывают показания прибора.

Существует множество типов вольтметров. Мы остановимся на аналоговых вольтметрах с магнитоэлектрическими измерительными механизмами. Эти механизмы довольно часто применяют в щитовых вольтметрах и многофункциональных измерительных приборах – мультиметрах. Магнитоэлектрический электрический механизм представляет собой проволочную катушку, размещенную между полюсами магнита.

Катушка подвешивается на спиральных пружинах обеспечивающих высокую чувствительность прибора. С катушкой связана указательная стрелка, с помощью которой осуществляется отсчет показаний на шкале прибора. Ниже на рисунке показано устройство магнитоэлектрического механизма.


Магнитоэлектрические измерительные механизмы имеют высокую чувствительность. С их помощью можно измерить напряжения составляющие сотые доли вольта. Для расширения пределов измерения последовательно с измерительным механизмом включают добавочные сопротивления. Схема простейшего вольтметра постоянного тока показана на рисунке.


Одним из важнейших параметром вольтметра является его внутреннее сопротивление. Чем больше значение внутреннего сопротивления вольтметра, тем меньшую погрешность можно получить в процессе измерения. Для аналоговых вольтметров внутреннее сопротивление обычно составляет 20кОм на вольт. Если необходимо получить большее значение сопротивления для измерений применяют электронные вольтметры, цифровые или аналоговые.

Для измерения переменного напряжения в конструкцию вольтметров включают выпрямители, которые преобразуют переменное напряжение в постоянное. Шкалы вольтметров для измерения переменного напряжения обычно градуируют в действующих (эффективных) значениях напряжения. Действующее значение переменного тока связано с максимальным следующим соотношением.

U=1/√2 U_m=0,707U_m (3)

Действующее значение удобно применять при вычислении мощности электрической цепи. Когда мы говорим, что в электрической розетке присутствует напряжение 220В, речь идет именно о действующем значении напряжения.

В коротком материале трудно рассказать обо всех нюансах связанных с электрическим напряжением и способах его измерения. Но мы надеемся, что текст окажется полезен читателю.

Ток, напряжение, сопротивление

Электрический ток ( I ) – это упорядоченное движение заряженных частиц. Первая мысль, которая приходит в голову из школьного курса физики – движение электронов. Безусловно.

Однако электрический заряд могут переносить не только они, а, например, еще ионы, определяющие возникновение электрического тока в жидкостях и газах.

Хочу предостеречь также от сравнения тока с протеканием воды по шлангу. (Хотя при рассмотрении Закона Кирхгофа такая аналогия будет уместна). Если каждая конкретная частица воды проделывает путь от начала до конца, то носитель электрического тока так не поступает.

Если уж нужна наглядность, то я бы привел пример переполненного автобуса, когда на остановке некто, втискиваясь в заднюю дверь, становится причиной выпадения из передней менее удачливого пассажира.

Условиями возникновения и существования электрического тока являются:

  • Наличие свободных носителей заряда
  • Наличие электрического поля, создающего и поддерживающего ток.

Будем считать, что теперь про электрический ток Вы знаете все. Это, конечно, шутка. Тем более что еще ничего не сказано про электрическое поле, которое у многих ассоциируется с напряжением, что не верно.

Электрическое поле – это вид материи, существующей вокруг электрически заряженных тел и оказывающее на них силовое воздействие. Опять же, обращаясь к знакомому со школы “одноименные заряды отталкиваются, а разноименные притягиваются” можно представить электрическое поле как нечто это воздействие передающее.

Это поле, равно как любое другое непосредственно ощутить нельзя, но существует его количественная характеристика – напряженность электрического поля.

Существует множество формул, описывающих взаимосвязь электрического поля с другими электрическими величинами и параметрами. Я ограничусь одной, сведенной к примитиву: E=Δφ.

Здесь:

  • E – напряженность электрического поля. Вообще это величина векторная, но я упростил все до скаляра.
  • Δφ=φ1-φ2 – разность потенциалов (рисунок 1).

Поскольку условием существования тока является наличие электрического поля, то его (поле) надо каким либо образом создать. Хорошо знакомые опыты электризации расчески, натирания тканью эбонитовой палочки, верчения ручки электростатической машины по вполне очевидным причинам на практике неприемлимы.

Поэтому были изобретены устройства, способные обеспечивать разность потенциалов за счет сил неэлектростатического происхождения (одно из них – хорошо всем известная батарейка), получившие название источник электродвижущей силы (ЭДС), которая обозначается так: ε.

Физический смысл ЭДС определяется работой, которую совершают сторонние силы, перемещая единичный заряд, но для того, чтобы получить первоначальное понятие что такое электрический ток, напряжение и сопротивление нам не нужно подробное рассмотрение этих процессов в интегральной и иных не менее сложных формах.

Напряжение ( U ).

Наотрез отказываюсь продолжать заморачивать Вам голову сугубо теоретическими выкладками и даю определение напряжения как разности потенциалов на участке цепи: U=Δφ=φ1-φ2, а для замкнутой цепи будем считать напряжение равным ЭДС источника тока: U=ε.

Это не совсем корректно, но на практике вполне достаточно.

Сопротивление ( R ) – название говорит само за себя – физическая величина, характеризующая противодействие проводника электрическому току. Формула, определяющая зависимость напряжения, тока и сопротивления называется закон Ома. Этот закон рассматривется на отдельной странице этого раздела.

Кроме того, сопротивление зависит от ряда факторов, например, материала проводника. Данные эти справочные, приводятся в виде значения удельного сопротивления ρ, определяемого как сопротивление 1 метра проводника/сечение. Чем меньше удельное сопротивление, тем меньше потери тока в проводнике.

Соответственно сопротивление проводника длиной L и площадью сечения S, будет составлять R=ρ*L/S.

Непосредственно из приведенной формулы видно, что сопротивление проводника также зависит от его длины и сечения. Температура тоже оказывает влияние на сопротивление.

Несколько слов про единицы измерения тока, напряжения, сопротивления. Основные единицы измерения этих величин следующие:

Ток – Ампер (А)
Напряжение – Вольт (В)
Сопротивление – Ом (Ом).

Это единицы измерения интернациональной системы (СИ) не всегда удобны. На практике применяются из производные (милиампер, килоом и пр.). При расчетах следует учитывать размерность всех величин, содержащихся в формуле. Так, если Вы, в законе Ома умножите ампер на килоом, то напряжение получите совсем не вольтах.

© 2012-2020 г. Все права защищены.

Представленные на сайте материалы имеют информационный характер и не могут быть использованы в качестве руководящих и нормативных документов


Напряжение, как его понизить и повысить

Электрическое напряжение между точками A и B электрической цепи или электрического поля — физическая величина, значение которой равно работе эффективного электрического поля (включающего сторонние поля), совершаемой при переносе единичного пробного электрического заряда из точки A в точку B.

Напряжение и сила тока – две основных величины в электричестве. Кроме них выделяют и ряд других величин: заряд, напряженность магнитного поля, напряженность электрического поля, магнитная индукция и другие. Практикующему электрику или электронщику в повседневной работе чаще всего приходится оперировать именно напряжением и током – Вольтами и Амперами. В этой статье мы расскажем именно о напряжении, о том, что это такое и как с ним работать.

Определение физической величины

Напряжение это разность потенциалов между двумя точками, характеризует выполненную работу электрического поля по переносу заряда из первой точки во вторую. Измеряется напряжение в Вольтах. Значит, напряжение может присутствовать только между двумя точками пространства. Следовательно, измерить напряжение в одной точке нельзя.

Потенциал обозначается буквой “Ф”, а напряжение буквой “U”. Если выразить через разность потенциалов, напряжение равно:

U=Ф1-Ф2

Если выразить через работу, тогда:

U=A/q,

где A – работа, q – заряд.

Измерение напряжения

Напряжение измеряется с помощью вольтметра. Щупы вольтметра подключают на две точки напряжение, между которыми нас интересует, или на выводы детали, падение напряжения на которой мы хотим измерить. При этом любое подключение к схеме может влиять на её работу. Это значит, что при добавлении параллельно элементу какой-либо нагрузки ток в цепи изменить и напряжение на элементе измениться по закону Ома.

Вывод:

Вольтметр должен обладать максимально высоким входным сопротивлением, чтобы при его подключении итоговое сопротивление на измеряемом участке оставалось практически неизменным. Сопротивление вольтметра должно стремиться к бесконечности, и чем оно больше, тем большая достоверность показаний.

На точность измерений (класс точности) влияет целый ряд параметров. Для стрелочных приборов – это и точность градуировки измерительной шкалы, конструктивные особенности подвеса стрелки, качество и целостность электромагнитной катушки, состояние возвратных пружин, точность подбора шунта и прочее.

Для цифровых приборов – в основном точность подбора резисторов в измерительном делителе напряжения, разрядность АЦП (чем больше, тем точнее), качество измерительных щупов.

Для измерения постоянного напряжения с помощью цифрового прибора (например, мультиметра), как правило, не имеет значения правильность подключения щупов к измеряемой цепи. Если вы подключите положительный щуп к точке с более отрицательным потенциалом, чем у точки, к которой подключен отрицательный щуп – то на дисплее перед результатом измерения появится знак “–”.

А вот если вы меряете стрелочным прибором нужно быть внимательным, При неправильном подсоединении щупов стрелка начнет отклоняться в сторону нуля, упрется в ограничитель. При измерении напряжений близких к пределу измерений или больше она может заклинить или погнуться, после чего о точности и дальнейшей работе этого прибора говорить не приходится.

Для большинства измерений в быту и в электронике на любительском уровне достаточно и вольтметра встроенного в мультиметры типа DT-830 и подобных.

Чем больше измеряемые значения – тем ниже требования к точности, ведь если вы измеряете доли вольта и у вас погрешность в 0. 1В – это существенно исказит картину, а если вы измеряете сотни или тысяч вольт, то погрешность и в 5 вольт не сыграет существенной роли.

Что делать если напряжение не подходит для питания нагрузки

Для питания каждого конкретного устройства или аппарата нужно подать напряжение определенной величины, но случается, так что имеющийся у вас источник питания не подходит и выдает низкое или слишком высокое напряжение. Решается эта проблема разными способами, в зависимости от требуемой мощности, напряжения и силы тока.

Как понизить напряжение сопротивлением?

Сопротивление ограничивает ток и при его протекании падает напряжение на сопротивление (токоограничивающий резистор). Такой способ позволяет понизить напряжение для питания маломощных устройств с токами потребления в десятки, максимум сотни миллиампер.

Примером такого питания можно выделить включение светодиода в сеть постоянного тока 12 (например, бортовая сеть автомобиля до 14. 7 Вольт). Тогда, если светодиод рассчитан на питание от 3.3 В, током в 20 мА, нужен резистор R:

R=(14.7-3.3)/0.02)= 570 Ом

Но резисторы отличаются по максимальной рассеиваемой мощности:

P=(14.7-3.3)*0.02=0.228 Вт

Ближайший по номиналу в большую сторону – резистор на 0.25 Вт.

Именно рассеиваемая мощность и накладывает ограничение на такой способ питания, обычно мощность резисторов не превышает 5-10 Вт. Получается, что если нужно погасить большое напряжение или запитать таким образом нагрузку мощнее, придется ставить несколько резисторов т.к. мощности одного не хватит и ее можно распределить между несколькими.

Способ снижения напряжения резистором работает и в цепях постоянного тока и в цепях переменного тока.

Недостаток – выходное напряжение ничем нестабилизировано и при увеличении и снижении тока оно изменяется пропорционально номиналу резистора.

Как понизить переменное напряжение дросселем или конденсатором?

Если речь вести только о переменном токе, то можно использовать реактивное сопротивление. Реактивное сопротивление есть только в цепях переменного тока, это связно с особенностями накопления энергии в конденсаторах и катушках индуктивности и законами коммутации.

Дроссель и конденсатор в переменном токе могут быть использованы в роли балластного сопротивления.

Реактивное сопротивление дросселя (и любого индуктивного элемента) зависит от частоты переменного тока (для бытовой электросети 50 Гц) и индуктивности, оно рассчитывается по формуле:

где ω – угловая частота в рад/с, L-индуктивность, 2пи – необходимо для перевода угловой частоты в обычную, f – частота напряжения в Гц.

Реактивное сопротивление конденсатора зависит от его емкости (чем меньше С, тем больше сопротивление) и частоты тока в цепи (чем больше частота, тем меньше сопротивление). Его можно рассчитать так:

Пример использования индуктивного сопротивление – это питание люминесцентных ламп освещения, ДРЛ ламп и ДНаТ. Дроссель ограничивает ток через лампу, в ЛЛ и ДНаТ лампах он используется в паре со стартером или импульсным зажигающем устройством (пусковое реле) для формирования всплеска высокого напряжения включающего лампу. Это связано с природой и принципом работы таких светильников.

А конденсатор используют для питания маломощных устройств, его устанавливают последовательно с питаемой цепью. Такой блок питания называется “бестрансфоматорный блок питания с балластным (гасящим) конденсатором”.

Очень часто встречают в качестве ограничителя тока заряда аккумуляторов (например, свинцовых) в носимых фонарях и маломощных радиоприемниках. Недостатки такой схемы очевидны – нет контроля уровня заряда аккумулятора, их выкипание, недозаряд, нестабильность напряжения.

Как понизить и стабилизировать напряжение постоянного тока

Чтобы добиться стабильного выходного напряжения можно использовать параметрические и линейные стабилизаторы. Часто их делают на отечественных микросхемах типа КРЕН или зарубежных типа L78xx, L79xx.

Линейный преобразователь LM317 позволяет стабилизировать любое значение напряжения, он регулируемый до 37В, вы можете сделать простейший регулируемый блок питания на его основе.

Если нужно незначительно снизить напряжение и стабилизировать его описанные ИМС не подойдут. Чтобы они работали должна быть разница порядка 2В и более. Для этого созданы LDO(low dropout)-стабилизаторы. Их отличие заключается в том, что для стабилизации выходного напряжение нужно, чтобы входное его превышало на величину от 1В. Пример такого стабилизатора AMS1117, выпускается в версиях от 1.2 до 5В, чаще всего используют версии на 5 и 3.3В, например в платах Arduino и многом другом.

Конструкция всех вышеописанных линейных понижающих стабилизаторов последовательного типа имеет существенный недостаток – низкий КПД. Чем больше разница между входным и выходным напряжением – тем он ниже. Он просто «сжигает» лишнее напряжение, переводя его в тепло, а потери энергии равны:

Pпотерь = (Uвх-Uвых)*I

Компания AMTECH выпускает ШИМ аналоги преобразователей типа L78xx, они работают по принципу широтно-импульсной модуляции и их КПД равен всегда более 90%.

Они просто включают и выключают напряжение с частотой до 300 кГц (пульсации минимальны). А действующее напряжение стабилизируется на нужном уровне. А схема включения аналогичная линейным аналогам.

Как повысить постоянное напряжение?

Для повышения напряжения производят импульсные преобразователи напряжения. Они могут быть включены и по схеме повышения (boost), и понижения (buck), и по повышающе-понижающей (buck-boost) схеме. Давайте рассмотрим несколько представителей:

1. Плата на базе микросхемы XL6009

2. Плата на базе LM2577, работает на повышение и понижение выходного напряжения.

3. Плата преобразователь на FP6291, подходит для сборки 5 V источника питания, например powerbank. С помощью корректировке номиналов резисторов может перестраиваться на другие напряжения, как и любые другие подобные преобразователь – нужно корректировать цепи обратной связи.

4. Плата на базе MT3608

Здесь всё подписано на плате – площадки для пайки входного – IN и выходного – OUT напряжения. Платы могут иметь регулировку выходного напряжения, а в некоторых случая и ограничения тока, что позволяет сделать простой и эффективный лабораторный блок питания. Большинство преобразователей, как линейных, так и импульсных имеют защиту от КЗ.

Как повысить переменное напряжение?

Для корректировки переменного напряжения используют два основных способа:

1. Автотрансформатор;

2. Трансформатор.

Автотрансформатор – это дроссель с одной обмоткой. Обмотка имеет отвод от определенного количества витков, так подключаясь между одним из концов обмотки и отводом, на концах обмотки вы получаете повышенное напряжение во столько раз, во сколько соотносится общее количество витков и количество витков до отвода.

Промышленностью выпускаются ЛАТРы – лабораторные автотрансформаторы, специальные электромеханические устройства для регулировки напряжения. Очень широко применение они нашли в разработке электронных устройств и ремонте источников питания. Регулировка достигается за счет скользящего щеточного контакта, к которому подключается питаемое устройство.

Недостатком таких устройств является отсутствие гальванической развязки. Это значит, что на выходных клеммах может запросто оказаться высокое напряжение, отсюда опасность поражения электрическим током.

Трансформатор – это классический способ изменения величины напряжения. Здесь есть гальваническая развязка от сети, что повышает безопасность таких установок. Величина напряжения на вторичной обмотке зависит от напряжений на первичной обмотки и коэффициента трансформации.

Uвт=Uперв*Kтр

Kтр=N1/N2

Отдельный вид – это импульсные трансформаторы. Они работают на высоких частотах в десятки и сотни кГц. Используются в подавляющем большинстве импульсных блоках питания, например:

– Зарядное устройство вашего смартфона;

– Блок питания ноутбука;

– Блок питания компьютера.

За счет работы на большой частоте снижаются массогабаритные показатели, они в разы меньше чем у сетевых (50/60 Гц) трансформаторов, количество витков на обмотках и, как следствие, цена. Переход на импульсные блоки питания позволил уменьшить габариты и вес всей современной электроники, снизить её потребление за счет увеличения кпд (в импульсных схемах 70-98%).

В магазинах часто встречаются электронные траснформаторы, на их вход подаётся сетевое напряжение 220В, а на выходе например 12 В переменное высокочастотное, для использования в нагрузке которая питается от постоянного тока нужно дополнительно устанавливать на выход диодный мост из высокоскоростных диодов.

Внутри находится импульсный трансформатор, транзисторные ключи, драйвер, или автогенераторная схема, как изображена ниже.

Достоинства – простота схемы, гальваническая развязка и малые размеры.

Недостатки – большинство моделей, что встречаются в продаже, имеют обратную связь по току, это значит что без нагрузки с минимальной мощностью (указано в спецификациях конкретного прибора) он просто не включится. Отдельные экземпляры оборудованы уже ОС по напряжению и работают на холостом ходу без проблем.

Используются чаще всего для питания 12В галогенных ламп, например точечные светильники подвесного потолка.

Заключение

Мы рассмотрели базовые сведения о напряжении, его измерении, а также регулировки. Современная элементная база и ассортимент готовых блоков и преобразователей позволяет реализовывать любые источники питания с необходимыми выходными характеристиками. Подробнее о каждом из способов можно написать отдельную статью, в пределах этой я постарался уместить базовые сведения, необходимые для быстрого подбора удобного для вас решения.

Ранее ЭлектроВести писали о топе-5 самых безумных батарей будущего

По материалам: electrik.info.

 

Как найти работу тока формула

Сегодня электрический ток имеет большую область применения. Связано это с тем, что он переносит с собой энергию, которую можно превратить в любую форму.

Что такое работа тока

При хаотичном движении заряженных частиц в проводнике электрическое поле будет совершать работу, которую решили назвать работой тока. Определение работы тока следующее: это работа электрического поля по переносу зарядов внутри проводника.

Важно! Помимо электрических сил, на проводник действуют еще и магнитные, которые также могут совершать работу. Однако в обычных условиях она будет очень мала.

Мощность

Абсолютно каждый электрический прибор рассчитан на поглощение энергии за единицу времени. Поэтому на практике большее значение имеет такое понятие, как мощность. Мощность — это скалярная физическая величина, в общем виде равная скорости изменения, преобразования, передачи или потребления энергии системы.

Единицы измерения

Любая физическая величина, которая может быть превращена в энергию, будет измеряться в Джоулях (Дж). 1 Джоуль равен работе при перемещении точки, к которой приложена сила, равная 1 Ньютону, умноженному на Путь в 1 метр. Получается, что 1 Дж = 1 Н · 1 м.

Единица измерения мощности — это Ватт (Вт). Он равен работе 1 Дж, совершенной за единицу времени в 1 с. Таким образом, 1 Вт = 1 Дж : 1 с

Формула вычисления

В 1841 году английский ученый Джеймс Джоуль сформулировал закон для нахождения количественной меры теплового воздействия электрического тока. В 1842 году этот же закон был также открыт русским физиком Эмилием Ленцем. Из-за этого он получил двойное название закона Джоуля-Ленца. В общем виде закон записывается следующим образом: Q = I² • R • t.

Он имеет достаточно обобщенный характер, так как не имеет зависимости от природных сил, генерирующих ток. Сегодня этот закон активно применяется в быту. Например, для определения степени нагрева вольфрамовой нити, используемой в лампочках.

Закон Джоуля-Ленца определяет количество теплоты, выделяемое током. Но, тем не менее, это поможет узнать, по каким формулам вычисляется работа электрического поля. Всё потому, что она впоследствии проявляется в виде нагревания проводника. Это говорит о том, что работа тока равна теплоте нагревания проводника (A=Q). Работа эл тока, формула: А= I² • R • t. Это не единственная формула для нахождения работы. Если использовать закон Ома для участка цепи (I=U:R), то можно вывести еще две формулы: А=I•U•t или A=U²:R.

Общая формула для того, чтобы вычислять мощность, заключается в ее прямой пропорциональности работе и обратной зависимости от времени (P=A:t). Если говорить о мощности в электрическом поле, то исходя из предыдущих формул, можно составить целых три: Р= I² • R; Р=I•U; Р=U²:R.

Приборы для измерения тока

Электроизмерительные приборы — это особый вид устройств, которые используются для измерения многих электрических величин. К ним относятся:

  • Амперметр переменного тока;
  • Вольтметр переменного тока;
  • Омметр;
  • Мультиметр;
  • Частометр;
  • Электрические счетчики.

Амперметр

Чтобы определить силу тока в электрической цепи, необходимо применить амперметр. Данный прибор включается в цепь последовательным образом и из-за пренебрежимо малого внутреннего сопротивления не оказывает влияния на ее состояние. Шкала амперметра проградуирована в амперах.

В классическом приборе через электромагнитную катушку проходит измеряемый ток, который образует магнитное поле, заставляющее отклоняться магнитную стрелку. Угол отклонения прямо пропорционален измеряемому току.

Электродинамический амперметр имеет более сложный принцип работы. В нем находятся две катушки: одна подвижная, другая стоит на месте. Между собой они могут быть соединены последовательно или параллельно. При прохождении тока через катушки их магнитные поля начинают взаимодействовать, что в результате заставляет подвижную катушку с закрепленной на ней стрелкой отклониться на некоторый угол, пропорциональный величине измеряемого тока.

Вольтметр

Для определения величины напряжения (разности потенциалов) на участке цепи используют вольтметр. Подключаться прибор должен параллельно цепи и обладать высоким внутренним сопротивлением. Тогда лишь сотые доли силы тока попадут в прибор.

Принцип работы заключается в том, что внутри вольтметра установлена катушка и последовательно подключенный резистор с сопротивлением не менее 1кОм, на котором проградуирована шкала вольтов. Самое интересное, что на самом деле резистор регистрирует силу тока. Однако деления подобраны таким образом, что показания соответствуют значению напряжения.

Омметр

Данный прибор используют для определения электрически активного сопротивления. Принцип действия состоит в изменении измеряемого сопротивления в напрямую зависящее от него напряжение благодаря операционному усилителю. Нужный объект должен быть подключен к цепи обратной связи или к усилителю.

Если омметр электронный, то он будет работать по принципу измерения силы тока, протекающего через необходимое сопротивление при постоянной разности потенциалов. Все элементы соединяют последовательно. В этом случае сила тока будет иметь следующую зависимость: I = U/(r0 + rx), где U — ЭДС источника, r0 — сопротивление амперметра, rx — искомое сопротивление. Согласно этой зависимости и определяют сопротивление.

Мультиметр

Приведенные в пример приборы сегодня используют лишь в школах на уроках физики. Для профессиональных задач были придуманы мультиметры. Самое обычное устройство включает в себя одновременно функции амперметра, вольтметра и омметра. Прибор бывает как легко переносимым, так и огромным стационарным с большим количеством возможностей. Название «мультиметр» в первый раз было применено именно к цифровому измерителю. Аналоговые приборы чаще называют «авометр», «тестер» или просто «Цешка».

Работа тока — сложная, но очень важная тема в электродинамике. Не зная ее, не получится решить даже простейших задач. Даже электрики используют формулы по нахождению работы для проведения необходимых подсчетов.

Чтобы подсчитать работу электрического тока, вспомним определение понятия напряжения: U=А/q

Следовательно, работа электрического тока равна:

Электрический заряд можно выразить через силу тока и его время протекания q=It:

Итак, работа электрического тока равна произведению силы тока на напряжение и на время протекания тока по цепи.

Работа электрического тока выражается в джоулях (Дж) . В качестве внесистемной единицы принята работа тока силой 1 А в течение 1 ч на участке цепи с напряжением 1 В. Эту единицу работы назвали ватт-час (1 Вт-ч) : 1 Вт-ч = 3600 Дж = 3,6 кДж. На практике используют более крупные, кратные ей единицы:

1 гВт-ч= 102 Вт-ч = 3,6·105Дж,
1 кВт-ч= 103 Вт-ч = 3,6·106Дж,
1 МВт-ч = 106 Вт-ч = 3,6·109Дж.

Из курса физики VII класса вы знаете, что мощность равна отношению совершенной работы ко времени, в течение которого эта работа была совершена. Мощность в механике принято обозначать буквой N, в электротехнике — буквой Р. Следовательно, мощность равна:

Пользуясь этой формулой, найдем мощность электрического тока. Так как работа тока определяется формулой А = IUt, то мощность электрического тока равна:

За единицу мощности ватт (Вт) принята мощность тока силой 1 А на участке с напряжением 1 В. Следовательно, 1 Вт = 1 А·1 В.

Ватт сравнительно небольшая мощность, на практике используют более крупные единицы, кратные ватту: 1 гВт (гектоватт) = 102 Вт, 1 кВт (киловатт) = 103 Вт, 1 МВт (мегаватт) = 106 Вт, 1 ГВт (гигаватт) = 109 Вт.

Этот видеоурок доступен по абонементу

У вас уже есть абонемент? Войти

На данном уроке мы рассмотрим вольтметр и амперметр, узнаем, что такое работа электрического тока и как она вычисляется.

Работа электрического тока

Сам по себе электрический ток не нужен. Важным является не сам ток, а его действие.

Действие электрического тока характеризуется работой электрического тока.

Работа – это величина, которая характеризует превращение энергии из одного вида в другой.

Например, была энергия кинетическая, стала энергия потенциальная, т. е. тело находилось в состоянии движения, затем оно остановилось, поднявшись при этом на некоторую высоту.

Что касается электрического тока, то мы уже знаем о движении электрических зарядов по проводнику и что движение это происходит под действием электрического поля, т. е. работу совершает электрическое поле. И работа в данном случае показывает, как энергия одного вида, например, энергия электрического тока, будет превращаться в другие виды энергии – механическую, тепловую и т. д.

Вывод формулы для нахождения работы электрического тока

Работа электрического тока связана, в первую очередь, с понятием электрического напряжения и силы тока.

Работа электрического поля – это произведение электрического напряжения на заряд, протекающий по проводнику.

Это утверждение получено из соотношения для электрического напряжения.

Электрическое напряжение – это работа электрического поля по переносу электрического заряда q.

Заряд – это есть произведение силы тока на время, в течение которого этот заряд протекает по проводнику.

Это утверждение следует из соотношения для силы тока.

Сила тока – это отношение заряда ко времени, в течение которого протекает заряд по проводнику через поперечное сечение проводника.

Подставив в формулу определения работы , получим выражение для вычисления работы электрического тока, работы электрического поля по перемещению электрического заряда.

Единицы измерения

Работа – 1 Джоуль или 1 Дж;

Напряжение – 1 Вольт или 1 В;

Сила тока – 1 Ампер или 1 А;

Время – 1 секунда или 1 с.

Определение

Работа электрического тока равна произведению силы тока на участке цепи, напряжению на концах этого участка и времени, в течение которого протекает ток по проводнику.

Приборы для измерения силы тока и электрического напряжения

Работа электрического тока связана с приборами, позволяющими определять значения указанных величин.

Напряжение определяется по прибору, который называется вольтметр. А для измерения силы тока используют амперметр (рис. 1).

Рис. 1. Изображения вольтметра и амперметра

Включив эти два прибора в электрическую цепь, наблюдая за показаниями этих приборов, определив время, в течение которого производятся измерения, определяем значение работы электрического тока. .

Заключение

Обратите внимание на то, что плата, которую мы производим за электроэнергию, – это плата именно за работу электрического тока. Действие электрического тока – это те самые действия, которые используются в технике, такой как нагревательные устройства, устройства, которые используются в быту (телевизоры, радиоприемники и т. д.).

Работа измеряется при помощи амперметра и вольтметра, но, тем не менее, есть отдельный прибор, который сразу способен измерять работу электрического тока

На следующем уроке мы познакомимся с понятием мощности.

Список литературы

  1. Генденштейн Л.Э, Кайдалов А.Б., Кожевников В.Б. / Под ред. Орлова В.А., Ройзена И.И. Физика 8. – М.: Мнемозина.
  2. Перышкин А.В. Физика 8. – М.: Дрофа, 2010.
  3. Фадеева А.А., Засов А.В., Киселев Д.Ф. Физика 8. – М.: Просвещение.

Дополнительные рекомендованные ссылки на ресурсы сети Интернет

Домашнее задание

  1. П. 50, вопросы 1–4, стр. 119, задание 24 (1). Перышкин А.В. Физика 8. – М.: Дрофа, 2010.
  2. Через реостат с сопротивлением 5 Ом протекает ток си­лой 0,5 А. Нужно определить, какую работу произведет ток в течение 4 часов (14 400 сек.).
  3. С помощью каких приборов можно измерить работу электрического поля?

Если вы нашли ошибку или неработающую ссылку, пожалуйста, сообщите нам – сделайте свой вклад в развитие проекта.

“>

Формула и определение электрического напряжения в цепи в физике

Пробовали ли вы когда-нибудь надувать воздушные шарики на время? Один надувает быстро, а другой за это же время надувает гораздо меньше. Без сомнения, первый совершает большую работу, чем второй.

С источниками напряжения происходит точно так же. Чтобы обеспечить движение частиц в проводнике, надо совершить работу. И эту работу совершает источник. Работу источника характеризует напряжение. Чем оно больше, тем большую работу совершает источник, тем ярче будет гореть лампочка в цепи (при других одинаковых условиях).

Напряжение равно отношению работы электрического поля по перемещению зарядак величине перемещаемого заряда на участке цепи.

U=Aq, где (U) — напряжение, (A) — работа электрического поля, (q) — заряд.

Обрати внимание!

Единица измерения напряжения в системе СИ — [(U)] = (1) B (вольт).

(1) вольт равен электрическому напряжению на участке цепи, где при протекании заряда, равного (1) Кл, совершается работа, равная (1) Дж: (1) В (= 1) Дж/1 Кл.

Все видели надпись на домашних бытовых приборах «(220) В». Она означает, что на участке цепи совершается работа (220) Дж по перемещению заряда (1) Кл.

Кроме вольта, применяют дольные и кратные ему единицы — милливольт и киловольт.

(1) мВ (= 0,001) В, (1) кВ (= 1000) В или (1) В (= 1000) мВ, (1) В (= 0,001) кВ.

Для измерения напряжения используют прибор, который называется вольтметр.

Обозначаются все вольтметры латинской буквой (V), которая наносится на циферблат приборов и используется в схематическом изображении прибора.

В школьных условиях используются вольтметры, изображённые на рисунке:

 

Основными элементами вольтметра являются корпус, шкала, стрелка и клеммы. Клеммы обычно подписаны плюсом или минусом и для наглядности выделены разными цветами: красный — плюс, черный (синий) — минус. Сделано это с той целью, чтобы заведомо правильно подключать клеммы прибора к соответствующим проводам, подключённым к источнику.

Обрати внимание!

В отличие от амперметра, который включается в разрыв цепи последовательно, вольтметр включается в цепь параллельно.

Включая вольтметр в цепь постоянного тока, необходимо соблюдать полярность.

Сборку электрической цепи лучше начинать со всех элементов, кроме вольтметра, а его уже подключать в самом конце.

Вольтметры делятся на приборы постоянного тока и переменного тока.

Если прибор предназначен для цепей переменного тока, то на циферблате принято изображать волнистую линию. Если прибор предназначен для цепей постоянного тока, то линия будет прямой.

Вольтметр постоянного токаВольтметр переменного тока

Можно обратить внимание на клеммы прибора. Если указана полярность («(+)» и «(-)»), то это прибор для измерения постоянного напряжения.

Иногда используют буквы (AC/DC). В переводе с английского (AC) (alternating current) — переменный ток, а (DC) (direct current) — постоянный ток.

В цепь переменного тока включается вольтметр для измерения переменного тока. Он полярности не имеет.

Обрати внимание!

Для измерения напряжения можно использовать и мультиметр.

Перед измерением необходимо прочитать инструкцию, чтобы правильно подключить прибор.

  • Следует помнить, что высокое напряжение опасно.
  • Что будет с человеком, который окажется рядом с упавшим оголённым кабелем, находящимся под высоким напряжением?
  • Так как земля является проводником электрического тока, вокруг упавшего оголённого кабеля, находящегося под напряжением, может возникнуть опасное для человека шаговое напряжение.

При попадании под шаговое напряжение даже небольшого значения возникают непроизвольные судорожные сокращения мышц ног. Обычно человеку удаётся в такой ситуации своевременно выйти из опасной зоны.

Обрати внимание!

Однако нельзя выбегать оттуда огромными шагами, шаговое напряжение при этом только увеличится! Выходить надо обязательно быстро, но очень мелкими шагами или скачками на одной ноге!

Существует много знаков, предупреждающих о высоком напряжении. Вот некоторые из них.

   

Безопасным напряжением для человека считается напряжение (42) В в нормальных условиях и (12) В в условиях с повышенной опасностью (сырость, высокая температура, металлические полы и др.).

Источники:

Пёрышкин А.В. Физика, 8 класс// ДРОФА, 2013.

http://class-fizika.narod.ru/8_29.htmhttp://interneturok.ru/ru/school/physics/8-klass/belektricheskie-yavleniyab/elektricheskoe-napryazhenie

http://kamenskih3.narod.ru/untitled74.htm

Источник: https://www.yaklass.ru/p/fizika/8-klass/elektricheskie-iavleniia-12351/elektricheskoe-napriazhenie-voltmetr-12361/re-9effb98b-8fe9-4eb7-9964-54c153a18241

Электрическое напряжение: определение, формулы и как измеряется

В данной статье мы подробно разберем что такое напряжение, как просто его представить и измерить.

Определение

Напряжение — это электродвижущая сила, которая толкает свободные электроны от одного атома к другому в том же направлении.

В первые дни электричества напряжение было известно как электродвижущая сила (ЭДС). Именно поэтому в уравнениях, таких как закон Ома, напряжение представлено символом Е.

Алессандро Вольта

Единицей электрического потенциала является вольт, названный в честь Алессандро Вольта, итальянского физика, жившего между 1745 и 1827 годами.

Алессандро Вольта был одним из пионеров динамического электричества. Исследуя основные свойства электричества, он изобрел первую батарею и углубил понимание электричества.

Представление напряжения

Легче всего понять напряжении, представив давлении в трубе. При более высоком напряжении (давлении) будет течь более сильный ток. Хотя важно понимать, что напряжение (давление) может существовать без тока (потока), но ток не может существовать без напряжения (давления).

Напряжение часто называют разностью потенциалов, потому что между любыми двумя точками в цепи будет существовать разница в потенциальной энергии электронов. Когда электроны протекают через батарею, их потенциальная энергия увеличивается, но когда они протекают через лампочку, их потенциальная энергия будет уменьшаться, эта энергия покинет цепь в виде света и тепла.

Возьмите, например, обычную 1,5-вольтовую батарею AA, между двумя клеммами (+ и -) есть разность потенциалов 1,5 Вольт.

Напряжение или разность потенциалов — это просто измерение количества энергии (в джоулях) на единицу заряда (кулона). Например, в 1,5-вольтовой батарее AA каждый кулон (заряд) будет получать 1,5 вольт или джоулей энергии.

  • Напряжение = [Джоуль ÷ Кулон]
  • 1 вольт = 1 джоуль на кулон
  • 100 вольт = 100 джоулей на кулон
  • 1 кулон = 6 200 000 000 000 000 000 электронов (6,2 × 10 18 )

В чем измеряется напряжение

Мы измеряем напряжение в единицах «Вольт», которые обычно обозначаются просто буквой «V» на чертежах и технической литературе. Часто необходимо количественно определить величину напряжения, это делается в соответствии с единицами СИ, наиболее распространенные величины напряжения, которые вы увидите:

  • мегавольт (мВ)
  • киловольт (кВ)
  • вольт (В)
  • милливольт (мВ)
  • микровольт (мкВ)

Напряжение всегда измеряется в двух точках с помощью устройства, называемого вольтметром. Вольтметры являются либо цифровыми, либо аналоговыми, причем последний является наиболее точным.

Вольтметры обычно встроены в портативные цифровые мультиметровые устройства, как показано ниже, они являются распространенным и часто важным инструментом для любого электрика или инженера-электрика.

Обычно вы найдете аналоговые вольтметры на старых электрических панелях, таких как распределительные щиты и генераторы, но почти все новое оборудование будет поставляться с цифровыми счетчиками в качестве стандарта.

Портативный цифровой мультиметр с функцией вольтметра

На электрических схемах вы увидите устройства вольтметра, обозначенные буквой V внутри круга, как показано ниже:

Расчет напряжения

В электрических цепях напряжение может быть рассчитано в соответствии с треугольником Ома. Чтобы найти напряжение (V), просто умножьте ток (I) на сопротивление (R).

Напряжение (V) = ток (I) * сопротивление (R)

V = I *R

Пример

  1. Ток в цепи (I) = 10 АСопротивление цепи (R) = 2 Ом
  2. Напряжение (V) = 10 А * 2 Ом
  3. Ответ: V = 20В

Резюме

  • Напряжение — это сила, которая перемещает электроны от одного атома к другому
  • Напряжение также известно как разность потенциалов
  • Напряжение измеряется в единицах «вольт» (В)
  • Батареи увеличивают потенциальную энергию электронов
  • Лампочки и другие нагрузки уменьшают потенциальную энергию электронов
  • Напряжение измеряется с помощью вольтметра
  • Напряжение цепи можно рассчитать путем умножения тока и сопротивления

Источник: https://meanders.ru/naprjazhenie.shtml

Что такое Электрическое напряжение — Определение, измерение

Большинство людей в быту могут оперировать таким понятием как электрическое напряжение. Практически все знают, что бытовая розетка находится под напряжением 220В, а пальчиковая батарейка выдает напряжение всего в 1.5В.

При этом далеко не каждый человек, окончивший среднюю школу или даже технический ВУЗ в состоянии ответить, что же все-таки означает термин электрическое напряжение.

В этом материале мы постараемся ответить на этот вопрос, по возможности не прибегая к сложной математике.

Определение электрического напряжения

В учебниках по физике и электротехнике можно встретить разные определения электрического напряжения. Одно из них звучит следующим образом: электрическое напряжение между двумя точками пространства равно разности потенциалов электрического поля в этих точках. Математически это записывается так:

U=φ_a-φ_b (1).

Где U – электрическое напряжение, а φ_a и φ_b потенциалы электрического поля в точках A и B соответственно.

Если мы не знаем что такое потенциал электрического поля в точке, то приведенное выше определение мало проясняет вопрос, что же такое электрическое напряжение.

Под потенциалом электрического поля в точке понимают работу, по перемещению единичного заряда совершаемую электрическим полем из данной точки в точку с нулевым потенциалом.

На первый взгляд определение электрического потенциала кажется довольно сложным. Например, не совсем понятно, где находится точка с нулевым потенциалом.

 Для начала нужно запомнить, что электрический потенциал это работа по переносу единичного заряда. Если обратиться к формуле (1) то станет ясно, что электрическое напряжение не что иное, как разность двух работ. То есть электрическое напряжение, тоже есть работа.

Отсюда мы приходим ко второму определению. Электрическое напряжение численно равно работе по переносу единичного электрического заряда из точки А в точку В. При этом φ_a и φ_b это потенциальная энергия которой обладает единичный заряд в точках А и В соответственно.

Для лучшего понимания изложенного выше можно привести следующую аналогию. Любое тело, находящееся на некотором расстоянии от Земли обладает потенциальной энергией.

Для того чтобы поднять тело выше придется выполнить некоторую работу. Величина этой работы будет равна разности потенциальных энергий, которыми обладает тело на разной высоте.

Похожую картину мы наблюдаем, когда мы имеем дело с электрическим полем.

Что касается точки пространства, в которой электрический заряд обладает нулевым электрическим потенциалом, то в теории электричества эту точку можно выбрать произвольно. Связанно это с тем, что электрическое поле «потенциально».

Чтобы прояснить этот термин придется прибегнуть к высшей математике, а мы решили этого избежать. На практике специалисты в области электротехники в качестве точек с нулевым потенциалом часто выбирают поверхность Земли.

И многие измерения выполняют относительно нее.

Электрические поля могут быть постоянными (неизменными во времени) и переменными. Переменные электрические поля могут изменяться по различным математическим законам.

В технике чаще всего используются переменные электрические поля, которые изменяются по закону синуса.

В случае переменного электрического поля мгновенное значение разности потенциалов между двумя точками можно вычислить по следующей формуле:

u(t)=U_m  sin⁡〖(ωt)〗 (2).

Здесь u – мгновенное значение напряжения; Um – максимальное значение напряжения; ω – частота, t – время.

Измерение электрического напряжения

Электрическое напряжение измеряют с помощью вольтметров. Для измерения напряжения (разности потенциалов) на участке электрической цепи щупы вольтметра подключают к концам этого участка и по шкале считывают показания прибора.

Существует множество типов вольтметров. Мы остановимся на аналоговых вольтметрах с магнитоэлектрическими измерительными механизмами. Эти механизмы довольно часто применяют в щитовых вольтметрах и многофункциональных измерительных приборах – мультиметрах.

Магнитоэлектрический электрический механизм представляет собой проволочную катушку, размещенную между полюсами магнита. Катушка подвешивается на спиральных пружинах обеспечивающих высокую чувствительность прибора. С катушкой связана указательная стрелка, с помощью которой осуществляется отсчет показаний на шкале прибора.

Ниже на рисунке показано устройство магнитоэлектрического механизма.

Магнитоэлектрические измерительные механизмы имеют высокую чувствительность. С их помощью можно измерить напряжения составляющие сотые доли вольта. Для расширения пределов измерения последовательно с измерительным механизмом включают добавочные сопротивления. Схема простейшего вольтметра постоянного тока показана на рисунке.

Одним из важнейших параметром вольтметра является его внутреннее сопротивление. Чем больше значение внутреннего сопротивления вольтметра, тем меньшую погрешность можно получить в процессе измерения.

Для аналоговых вольтметров внутреннее сопротивление обычно составляет 20кОм на вольт.

Если необходимо получить большее значение сопротивления для измерений применяют электронные вольтметры, цифровые или аналоговые.

Для измерения переменного напряжения в конструкцию вольтметров включают выпрямители, которые преобразуют переменное напряжение в постоянное. Шкалы вольтметров для измерения переменного напряжения обычно градуируют в действующих (эффективных) значениях напряжения. Действующее значение переменного тока связано с максимальным следующим соотношением.

U=1/√2 U_m=0,707U_m (3)

Действующее значение удобно применять при вычислении мощности электрической цепи. Когда мы говорим, что в электрической розетке присутствует напряжение 220В, речь идет именно о действующем значении напряжения.

В коротком материале трудно рассказать обо всех нюансах связанных с электрическим напряжением и способах его измерения. Но мы надеемся, что текст окажется полезен читателю.

Источник: https://Elektrika.ru/articles/elektroprovodka/elektricheskoe_napryazhenie/

Электрическое напряжение цепи

При описании протекающих в электроцепи процессов в электротехнике применяют такие понятия, как сопротивление, напряжение и ток. Каждому из этих понятий свойственны свои специфические характеристики, и они имеют соответствующее назначение.

Обязательным для протекания зарядов требованием считается наличие цепи (замкнутого контура, обеспечивающего все необходимые условия для их передвижения). При формировании разрыва внутри движущихся частиц их направленное перемещение резко прекращается.

По такому принципу работают все типы выключателей и используемые в электрике защиты. Они осуществляют разделение между собой за счет подвижных контактов токопроводящих частей. Это действие и способствует прерыванию процесса протекания электрического тока после отключения электроприбора.

Понятие электрического напряжения в физике

Электрическим током в физике считается направленное перемещение заряженных частиц, создаваемое электрополем, совершающим при этом определенную работу.

Определение 1

Работа создающего ток электрополя называется работой тока ($A$). Такая работа может на разных участках цепи отличаться, однако при этом она будет пропорциональной проходящему через него заряду.

Ничего непонятно?

Попробуй обратиться за помощью к преподавателям

  • Физической величиной работы тока на конкретном участке при перемещении по нему заряда 1 Кл считается электрическое напряжение ($U$).
  • Для определения напряжения на отдельно взятом участке существует следующая формула:
  • $U =frac{A}{q}$, где:
  • $A$ — работа тока,
  • $q$ — прошедший по участку заряд.

Возникновение тока в электрической цепи

Замечание 1

Электрическую цепь характеризует комплекс устройств, обеспечивающих путь для протекающего электрического тока и соединенных определенным образом. В качестве элементов электроцепи служат: нагрузка, проводники и источник тока. В составе электрической цепи могут быть и другие элементы, как, например, устройства защиты и коммутации.

Необходимым условием возникновения тока будет соединение двух точек, у одной из которых очень много электронов в отличие от другой. Иными словами, потребуется образование разности потенциалов между указанными точками. С этой целью в цепи используется источник тока. Таким источником могут служить устройства в виде генераторов, батарей, химических элементов и др.

В качестве нагрузки в электроцепи выступает абсолютно любой потребитель электроэнергии. Нагрузка способна оказывать сопротивление электрическому току. От величины такого сопротивления будет зависеть величина тока. Ток течет по проводникам от источника тока к нагрузке. Проводниками, в свою очередь, служат материалы, имеющие наименьшее сопротивление, такие, как золото, серебро, медь.

Типы соединения элементов в электрической цепи

В электротехнике, в зависимости от типа соединения элементов электроцепи, существуют такие виды электрических цепей:

  • последовательная;
  • параллельная электрическая цепь;
  • последовательно-параллельная.

В электрической цепи последовательного типа соединении все элементы соединены друг с другом последовательно. Это означает, что конец первого элемента соединяется с началом второго и т.д.

  1. Для тока такое соединение элементов дает только один путь протекания от источника к нагрузке. Общий ток цепи при этом будет равен току, который проходит через каждый элемент цепи:
  2. $I_{общ} = I_1=I_2=I_3$
  3. При падающем напряжении вдоль всей цепи оно будет равняться приложенному к рассматриваемому участку (AB) напряжению $E$ и сумме падений напряжений на всех участках электроцепи (резисторах). Это выражает следующая формула:
  4. $E=U(A-B)=U_1+U_2+U_3$
  5. Элементы в параллельной электрической цепи соединены так, что начало каждого из них соединяется в одну общую точку, а концы при этом — в другую.
  6. Для тока в этом случае существует несколько путей протекания к нагрузкам от источника. При этом общий ток цепи $I_{общ}$ получен посредством формулы:
  7. $I_{общ}=I_1+I_2+I_3$
  8. Падение напряжения на всех резисторах выражает следующая формула: $E=U_1=U_2=U_3$

Последовательно-параллельная электроцепь представляет комбинацию цепи последовательного и параллельного типа соединения. Другими словами, ее элементы могут включаться, как последовательным, так и параллельным образом.

Электрическое напряжение в цепях постоянного, переменного и трехфазного тока

Определение 2

Напряжением в цепи постоянного тока на участке между точками A и B считается совершаемая электрическим полем работа в момент переноса пробного положительного заряда из первой точки во вторую.

При описании цепей переменного тока используют такие виды напряжений: мгновенное, амплитудное, среднее, среднеквадратичное.

Мгновенное напряжение представляет разность потенциалов двух точек, которая была измерена в конкретный момент времени. Данный вид напряжения будет зависеть от времени.

Амплитудным считается максимальное по модулю значение мгновенного напряжения, взятое за весь период колебаний:

$U_M=max(u(t))$

В цепях трехфазного тока существует напряжение фазного и линейного типа. Под фазным понимается среднеквадратичное значение напряжения на каждой отдельной фазе нагрузки. Линейным считается напряжение между подводящими фазными проводами. Если нагрузка соединяется в треугольник, фазное и линейное напряжение будут равны.

Источник: https://spravochnick.ru/fizika/napryazhenie_elektricheskogo_toka/elektricheskoe_napryazhenie_cepi/

Напряжение электрического тока и вольтметр

Электрический ток – это проходящие через проводник электроны, несущие отрицательный заряд. Объем этого заряда или, иными словами, количество электричества характеризует силу тока. Мы знаем, что сила тока одинакова во всех местах цепи.

Электроны не могут исчезать или «спрыгивать» с проводов и нагрузки. Поэтому, силу тока мы можем измерить в любом месте электрической цепи. Однако, будет ли одинаковым действие тока на разные участки этой цепи? Давайте разберемся.

Проходя по проводам, ток лишь слегка их нагревает, однако не совершает при этом большой работы.

Проходя же через спираль электрической лампочки, ток не просто сильно нагревает ее, он нагревает ее до такой степени, что она, раскаляясь, начинает светиться.

То есть в данном случае ток совершает механическую работу, и довольно приличную работу. Ток тратит свою энергию. Электроны в том же количестве продолжают бежать дальше, но энергии у них уже поменьше.

Определение электрического напряжения

То есть электрическое поле должно было «протащить» электроны через нагрузку, и энергия, которая при этом израсходовалась, характеризуется величиной, называемой электрическим напряжением.

Эта же энергия потратилась на какое-то изменение состояния вещества нагрузки. Энергия, как мы знаем, не пропадает в никуда и не появляется из ниоткуда. Об этом гласит Закон сохранения энергии.

То есть, если ток потратил энергию на прохождение через нагрузку, эту энергию приобрела нагрузка и, например, нагрелась.

То есть, приходим к определению: напряжение электрического тока – это величина, показывающая, какую работу совершило поле при перемещении заряда от одной точки до другой. Напряжение в разных участках цепи будет различным.

Напряжение на участке пустого провода будет совсем небольшим, а напряжение на участке с какой-либо нагрузкой будет гораздо большим, и зависеть величина напряжения будет от величины работы, произведенной током. Измеряют напряжение в вольтах (1 В).

Для определения напряжения существует формула: 

U=A/q,

где U — напряжение,A – работа, совершенная током по перемещению заряда q на некий участок цепи.

Напряжение на полюсах источника тока

Что касается напряжения на участке цепи – все понятно.

А что же тогда означает напряжение на полюсах источника тока? В данном случае это напряжение означает потенциальную величину энергии, которую может источник придать току. Это как давление воды в трубах.

Эта величина энергии, которая будет израсходована, если к источнику подключить некую нагрузку. Поэтому, чем большее напряжение у источника тока, тем большую работу может совершить ток.

Вольтметр

Для измерения напряжения существует прибор, называемый вольтметром. В отличие от амперметра, он подключается не произвольно в любом месте цепи, а параллельно нагрузке, до нее и после. В таком случае вольтметр показывает величину напряжения, приложенного к нагрузке. Для измерения напряжения на полюсах источника тока, вольтметр подключают непосредственно к полюсам прибора.

Нужна помощь в учебе?

Предыдущая тема: Сила тока: природа, формула, измерение амперметром
Следующая тема:   Сопротивление тока: притяжение ядер, проводники и непроводники

Источник: http://www.nado5.ru/e-book/ehlnapryazhenie-voltmetr

Постоянный электрический ток

Автор статьи — профессиональный репетитор, автор учебных пособий для подготовки к ЕГЭ Игорь Вячеславович Яковлев

Темы кодификатора ЕГЭ: постоянный электрический ток, сила тока, напряжение

Электрический ток обеспечивает комфортом жизнь современного человека. Технологические достижения цивилизации — энергетика, транспорт, радио, телевидение, компьютеры, мобильная связь — основаны на использовании электрического тока.

Электрический ток — это направленное движение заряженных частиц, при котором происходит перенос заряда из одних областей пространства в другие.

Электрический ток может возникать в самых различных средах: твёрдых телах, жидкостях, газах. Порой и среды никакой не нужно — ток может существовать даже в вакууме! Мы поговорим об этом в своё время, а пока приведём лишь некоторые примеры.

• Замкнём полюса батарейки металлическим проводом. Свободные электроны провода начнут направленное движение от «минуса» батарейки к «плюсу».
Это — пример тока в металлах.

• Бросим в стакан воды щепотку поваренной соли . Молекулы соли диссоциируют на ионы, так что в растворе появятся свободные заряды: положительные ионы и отрицательные ионы . Теперь засунем в воду два электрода, соединённые с полюсами батарейки. Ионы начнут направленное движение к отрицательному электроду, а ионы — к положительному.
Это — пример прохождения тока через раствор электролита.

• Грозовые тучи создают столь мощные электрические поля, что оказывается возможным пробой воздушного промежутка длиной в несколько километров. В результате сквозь воздух проходит гигантский разряд — молния.
Это — пример электрического тока в газе.

Во всех трёх рассмотренных примерах электрический ток обусловлен движением заряженных частиц внутри тела и называется током проводимости.

• Вот несколько иной пример. Будем перемещать в пространстве заряженное тело. Такая ситуация согласуется с определением тока! Направленное движение зарядов — есть, перенос заряда в пространстве — присутствует. Ток, созданный движением макроскопического заряженного тела, называется конвекционным.

Заметим, что не всякое движение заряженных частиц образует ток.

Например, хаотическое тепловое движение зарядов проводника — не направленное (оно совершается в каких угодно направлениях), и потому током не является (при возникновении тока свободные заряды продолжают совершать тепловое движение! Просто в этом случае к хаотическим перемещениям заряженных частиц добавляется их упорядоченный дрейф в определённом
направлении).

Не будет током и поступательное движение электрически нейтрального тела: хотя заряженные частицы в его атомах и совершают направленное движение, не происходит переноса заряда из одних участков пространства в другие.

Направление электрического тока

Направление движения заряженных частиц, образующих ток, зависит от знака их заряда. Положительно заряженные частицы будут двигаться от «плюса» к «минусу», а отрицательно заряженные — наоборот, от «минуса» к «плюсу».

В электролитах и газах, например, присутствуют как положительные, так и отрицательные свободные заряды, и ток создаётся их встречным движением в обоих направлениях.

Какое же из этих направлений принять за направление электрического тока?

Направлением тока принято считать направление движения положительных зарядов.

Попросту говоря, по соглашению ток течёт от «плюса» к «минусу» (рис. 1; положительная клемма источника тока изображена длинной чертой, отрицательная клемма — короткой).

Рис. 1. Направление тока

Данное соглашение вступает в некоторое противоречие с наиболее распространённым случаем металлических проводников. В металле носителями заряда являются свободные электроны, и двигаются они от «минуса» к «плюсу». Но в соответствии с соглашением мы вынуждены считать, что направление тока в металлическом проводнике противоположно движению свободных электронов. Это, конечно, не очень удобно.

Тут, однако, ничего не поделаешь — придётся принять эту ситуацию как данность. Так уж исторически сложилось.

Выбор направления тока был предложен Ампером (договорённость о направлении тока понадобилась Амперу для того, чтобы дать чёткое правило определения направления силы, действующей на проводник с током в магнитном поле.

Сегодня эту силу мы называем силой Ампера, направление которой определяется по правилу левой руки) в первой половине XIX века, за 70 лет до открытия электрона. К этому выбору все привыкли, и когда в 1916 году выяснилось, что ток в металлах вызван движением свободных электронов, ничего менять уже не стали.

Действия электрического тока

Как мы можем определить, протекает электрический ток или нет? О возникновении электрического тока можно судить по следующим его проявлениям.

1. Тепловое действие тока. Электрический ток вызывает нагревание вещества, в котором он протекает. Именно так нагреваются спирали нагревательных приборов и ламп накаливания. Именно поэтому мы видим молнию. В основе действия тепловых амперметров лежит тепловое расширение проводника с током, приводящее к перемещению стрелки прибора.

2. Магнитное действие тока. Электрический ток создаёт магнитное поле: стрелка компаса, расположенная рядом с проводом, при включении тока поворачивается перпендикулярно проводу.

Магнитное поле тока можно многократно усилить, если обмотать провод вокруг железного стержня — получится электромагнит.

На этом принципе основано действие амперметров магнитоэлектрической системы: электромагнит поворачивается в поле постоянного магнита, в результате чего стрелка прибора перемещается по шкале.

3. Химическое действие тока. При прохождении тока через электролиты можно наблюдать изменение химического состава вещества. Так, в растворе положительные ионы двигаются к отрицательному электроду, и этот электрод покрывается медью.

Электрический ток называется постоянным, если за равные промежутки времени через поперечное сечение проводника проходит одинаковый заряд.

Постоянный ток наиболее прост для изучения. С него мы и начинаем.

Сила и плотность тока

Количественной характеристикой электрического тока является сила тока. В случае постоянного тока абсолютная величина силы тока есть отношение абсолютной величины заряда , прошедшего через поперечное сечение проводника за время , к этому самому времени:

(1)

Измеряется сила тока в амперах (A). При силе тока в А через поперечное сечение проводника за с проходит заряд в Кл.

Подчеркнём, что формула (1) определяет абсолютную величину, или модуль силы тока.
Сила тока может иметь ещё и знак! Этот знак не связан со знаком зарядов, образующих ток, и выбирается из иных соображений.

А именно, в ряде ситуаций (например, если заранее не ясно, куда потечёт ток) удобно зафиксировать некоторое направление обхода цепи (скажем, против часовой стрелки) и считать силу тока положительной, если направление тока совпадает с направлением обхода, и отрицательной, если ток течёт против направления обхода (сравните с тригонометрическим кругом: углы считаются положительными, если отсчитываются против часовой стрелки, и отрицательными, если по часовой стрелке).

В случае постоянного тока сила тока есть величина постоянная. Она показывает, какой заряд проходит через поперечное сечение проводника за с.

  • Часто бывает удобно не связываться с площадью поперечного сечения и ввести величину плотности тока:
  • (2)
  • где — сила тока, — площадь поперечного сечения проводника (разумеется, это сечение перпендикулярно направлению тока). С учётом формулы (1) имеем также:

Плотность тока показывает, какой заряд проходит за единицу времени через единицу площади поперечного сечения проводника. Согласно формуле (2), плотность тока измеряется в А/м2.

Скорость направленного движения зарядов

Когда мы включаем в комнате свет, нам кажется, что лампочка загорается мгновенно. Скорость распространения тока по проводам очень велика: она близка к км/с (скорости света в вакууме). Если бы лампочка находилась на Луне, она зажглась бы через секунду с небольшим.

Однако не следует думать, что с такой грандиозной скоростью двигаются свободные заряды, образующие ток. Оказывается, их скорость составляет всего-навсего доли миллиметра в секунду.

Почему же ток распространяется по проводам так быстро? Дело в том, что свободные заряды взаимодействуют друг с другом и, находясь под действием электрического поля источника тока, при замыкании цепи приходят в движение почти одновременно вдоль всего проводника. Скорость распространения тока есть скорость передачи электрического взаимодействия между свободными зарядами, и она близка к скорости света в вакууме. Скорость же, с которой сами заряды перемещаются внутри проводника, может быть на много порядков меньше.

Итак, подчеркнём ещё раз, что мы различаем две скорости.

1. Скорость распространения тока. Это — скорость передачи электрического сигнала по цепи. Близка к км/с.

2. Скорость направленного движения свободных зарядов. Это — средняя скорость перемещения зарядов, образующих ток. Называется ещё скоростью дрейфа.

Мы сейчас выведем формулу, выражающую силу тока через скорость направленного движения зарядов проводника.

Пусть проводник имеет площадь поперечного сечения (рис. 2). Свободные заряды проводника будем считать положительными; величину свободного заряда обозначим (в наиболее важном для практики случая металлического проводника это есть заряд электрона). Концентрация свободных зарядов (т. е. их число в единице объёма) равна .

Рис. 2. К выводу формулы

  1. Какой заряд пройдёт через поперечное сечение нашего проводника за время ?
  2. С одной стороны, разумеется,
  3. (3)
  4. С другой стороны, сечение пересекут все те свободные заряды, которые спустя время окажутся внутри цилиндра с высотой . Их число равно:
  5. Следовательно, их общий заряд будет равен:
  6. (4)
  7. Приравнивая правые части формул (3) и (4) и сокращая на , получим:
  8. (5)
  9. Соответственно, плотность тока оказывается равна:
  10. Давайте в качестве примера посчитаем, какова скорость движения свободных электронов в медном проводе при силе тока A.
  11. Заряд электрона известен: Кл.

Чему равна концентрация свободных электронов? Она совпадает с концентрацией атомов меди, поскольку от каждого атома отщепляется по одному валентному электрону. Ну а концентрацию атомов мы находить умеем:

  • м
  • Положим мм . Из формулы (5) получим:
  • м/с.
  • Это порядка одной десятой миллиметра в секунду.

Стационарное электрическое поле

Мы всё время говорим о направленном движении зарядов, но ещё не касались вопроса о том, почему свободные заряды совершают такое движение. Почему, собственно, возникает электрический ток?

Для упорядоченного перемещения зарядов внутри проводника необходима сила, действующая на заряды в определённом направлении. Откуда берётся эта сила? Со стороны электрического поля!

Чтобы в проводнике протекал постоянный ток, внутри проводника должно существовать стационарное (то есть — постоянное, не зависящее от времени) электрическое поле. Иными словами, между концами проводника нужно поддерживать постоянную разность потенциалов.

Стационарное электрическое поле должно создаваться зарядами проводников, входящих в электрическую цепь. Однако заряженные проводники сами по себе не смогут обеспечить протекание постоянного тока.

Рассмотрим, к примеру, два проводящих шара, заряженных разноимённо. Соединим их проводом. Между концами провода возникнет разность потенциалов, а внутри провода — электрическое поле. По проводу потечёт ток.

Но по мере прохождения тока разность потенциалов между шарами будет уменьшаться, вслед за ней станет убывать и напряжённость поля в проводе. В конце концов потенциалы шаров станут равны друг другу, поле в проводе обратится в нуль, и ток исчезнет.

Мы оказались в электростатике: шары плюс провод образуют единый проводник, в каждой точке которого потенциал принимает одно и то же значение; напряжённость
поля внутри проводника равна нулю, никакого тока нет.

То, что электростатическое поле само по себе не годится на роль стационарного поля, создающего ток, ясно и из более общих соображений. Ведь электростатическое поле потенциально, его работа при перемещении заряда по замкнутому пути равна нулю. Следовательно, оно не может вызывать циркулирование зарядов по замкнутой электрической цепи — для этого требуется совершать ненулевую работу.

Кто же будет совершать эту ненулевую работу? Кто будет поддерживать в цепи разность потенциалов и обеспечивать стационарное электрическое поле, создающее ток в проводниках?

Ответ — источник тока, важнейший элемент электрической цепи.

Чтобы в проводнике протекал постоянный ток, концы проводника должны быть присоединены к клеммам источника тока (батарейки, аккумулятора и т. д.).

Клеммы источника — это заряженные проводники. Если цепь замкнута, то заряды с клемм перемещаются по цепи — как в рассмотренном выше примере с шарами. Но теперь разность потенциалов между клеммами не уменьшается: источник тока непрерывно восполняет заряды на клеммах, поддерживая разность потенциалов между концами цепи на неизменном уровне.

В этом и состоит предназначение источника постоянного тока. Внутри него протекают процессы неэлектрического (чаще всего — химического) происхождения, которые обеспечивают непрерывное разделение зарядов. Эти заряды поставляются на клеммы источника в необходимом количестве.

Количественную характеристику неэлектрических процессов разделения зарядов внутри источника — так называемую ЭДС — мы изучим позже, в соответствующем листке.

А сейчас вернёмся к стационарному электрическому полю. Каким же образом оно возникает в проводниках цепи при наличии источника тока?

Заряженные клеммы источника создают на концах проводника электрическое поле. Свободные заряды проводника, находящиеся вблизи клемм, приходят в движение и действуют своим электрическим полем на соседние заряды.

Со скоростью, близкой к скорости света, это взаимодействие передаётся вдоль всей цепи, и в цепи устанавливается постоянный электрический ток. Стабилизируется и электрическое поле, создаваемое движущимися зарядами.

Стационарное электрическое поле — это поле свободных зарядов проводника, совершающих направленное движение.

Стационарное электрическое поле не меняется со временем потому, что при постоянном токе не меняется картина распределения зарядов в проводнике: на место заряда, покинувшего данный участок проводника, в следующий момент времени поступает точно такой же заряд. По этой причине стационарное поле во многом (но не во всём) аналогично полю электростатическому.

А именно, справедливы следующие два утверждения, которые понадобятся нам в дальнейшем (их доказательство даётся в вузовском курсе физики).

1. Как и электростатическое поле, стационарное электрическое поле потенциально. Это позволяет говорить о разности потенциалов (т. е. напряжении) на любом участке цепи (именно эту разность потенциалов мы измеряем вольтметром).

Потенциальность, напомним, означает, что работа стационарного поля по перемещению заряда не зависит от формы траектории.

Именно поэтому при параллельном соединении проводников напряжение на каждом из них одинаково: оно равно разности потенциалов стационарного поля между теми двумя точками, к которым подключены проводники.
2.

В отличие от электростатического поля, стационарное поле движущихся зарядов проникает внутрь проводника (дело в том, что свободные заряды, участвуя в направленном движении, не успевают должным образом перестраиваться и принимать «электростатические» конфигурации).

Линии напряжённости стационарного поля внутри проводника параллельны его поверхности, как бы ни изгибался проводник. Поэтому, как и в однородном электростатическом поле, справедлива формула , где — напряжение на концах проводника, — напряжённость стационарного поля в проводнике, — длина проводника.

Источник: https://ege-study.ru/ru/ege/materialy/fizika/postoyannyj-elektricheskij-tok/

Электрическое напряжение — это… Что такое Электрическое напряжение?

Электри́ческое напряже́ние между точками A и B электрической цепи или электрического поля — физическая величина, значение которой равно отношению работы электрического поля, совершаемой при переносе пробного электрического заряда из точки A в точку B, к величине пробного заряда.

При этом считается, что перенос пробного заряда не изменяет распределения зарядов на источниках поля (по определению пробного заряда).

В потенциальном электрическом поле эта работа не зависит от пути, по которому перемещается заряд. В этом случае электрическое напряжение между двумя точками совпадает с разностью потенциалов между ними.

— интеграл от проекции поля эффективной напряжённости поля (включающего сторонние поля) на расстояние между точками A и B вдоль заданной траектории, идущей из точки A в точку B. В электростатическом поле значение этого интеграла не зависит от пути интегрирования и совпадает с разностью потенциалов.

Единицей измерения напряжения в системе СИ является вольт.

Напряжение в цепях постоянного тока

Напряжение в цепи постоянного тока определяется так же, как и в электростатике.

Напряжение в цепях переменного тока

Для описания цепей переменного тока применяются следующие понятия:

Мгновенное напряжение

Мгновенное напряжение есть разность потенциалов между двумя точками, измеренная в данный момент времени. Оно является функцией времени:

Амплитудное значение напряжения

Амплитуда напряжения есть максимальное по модулю значение мгновенного напряжения за весь период колебаний:

Для гармонических (синусоидальных) колебаний напряжения мгновенное значение напряжения выражается как:

Для сети переменного синусоидального напряжения со среднеквадратичным значением 220 В амплитудное равно приблизительно 311,127 В.

Амплитудное напряжение можно измерить с помощью осциллографа.

Среднее значение напряжения

Среднее значение напряжения (постоянная составляющая напряжения) определяется за весь период колебаний, как:

Для чистой синусоиды среднее значение напряжения равно нулю.

Среднеквадратичное значение напряжения

Среднеквадратичное значение (устаревшее наименование: действующее, эффективное) наиболее удобно для практических расчётов, так как на линейной активной нагрузке оно совершает ту же работу (например, лампа накаливания имеет ту же яркость свечения, нагревательный элемент выделяет столько же тепла), что и равное ему постоянное напряжение:

Для синусоидального напряжения справедливо равенство:

В технике и быту при использовании переменного тока под термином «напряжение» имеется в виду именно эта величина, и все вольтметры проградуированы исходя из её определения.

Однако конструктивно большинство приборов фактически измеряют не среднеквадратичное, а средневыпрямленное (см.

ниже) значение напряжения, поэтому для несинусоидального сигнала их показания могут отличаться от истинного значения.

Средневыпрямленное значение напряжения

Средневыпрямленное значение есть среднее значение модуля напряжения:

Для синусоидального напряжения справедливо равенство:

На практике используется редко, однако большинство вольтметров переменного тока (те, в которых ток перед измерением выпрямляется) фактически измеряют именно эту величину, хотя их шкала и проградуирована по среднеквадратичным значениям.

Напряжение в цепях трёхфазного тока

В цепях трёхфазного тока различают фазное и линейное напряжения.

Под фазным напряжением понимают среднеквадратичное значение напряжения на каждой из фаз нагрузки, а под линейным — напряжение между подводящими фазными проводами.

При соединении нагрузки в треугольник фазное напряжение равно линейному, а при соединении в звезду (при симметричной нагрузке или при глухозаземлённой нейтрали) линейное напряжение в раз больше фазного.

На практике напряжение трёхфазной сети обозначают дробью, в знаменателе которой стоит линейное напряжение, а в числителе — фазное при соединении в звезду (или, что то же самое, потенциал каждой из линий относительно земли). Так, в России наиболее распространены сети с напряжением 220/380 В; также иногда используются сети 127/220 В и 380/660 В.

Стандарты

Объект
Тип напряжения
Значение (на вводе потребителя)
Значение (на выходе источника)
ЭлектрокардиограммаИмпульсное1-2 мВ
Телевизионная антеннаПеременное высокочастотное1-100 мВ
Батарейка AA («пальчиковая»)Постоянное1,5 В
Литиевая батарейкаПостоянное3 В — 1,8 В (в исполнении пальчиковой батарейки , на примере Varta Professional Lithium, AA)
Управляющие сигналы компьютерных компонентовИмпульсное3,5 В, 5 В
Батарейка типа 6F22 («Крона»)Постоянное9 В
Силовое питание компьютерных компонентовПостоянное12 В
Электрооборудование автомобиляПостоянное12/24 В
Блок питания ноутбука и жидкокристаллических мониторовПостоянное19 В
Сеть «безопасного» пониженного напряжения для работы в опасных условияхПеременное36-42 В
Напряжение наиболее стабильного горения свечи ЯблочковаПостоянное55 В
Напряжение в телефонной линии (при опущенной трубке)Постоянное60 В
Напряжение в электросети ЯпонииПеременное трёхфазное100/172 В
Напряжение в домашних электросетях СШАПеременное трёхфазное120 В / 240 В (сплит-фаза)
Напряжение в электросети РоссииПеременное трёхфазное220/380 В230/400 В
Разряд электрического скатаПостоянноедо 200—250 В
Контактная сеть трамвая и троллейбусаПостоянное550 В600 В
Разряд электрического угряПостоянноедо 650 В
Контактная сеть метрополитенаПостоянное750 В825 В
Контактная сеть электрифицированной железной дороги (Россия, постоянный ток)Постоянное3 кВ3,3 кВ
Распределительная воздушная линия электропередачи небольшой мощностиПеременное трёхфазное6-20 кВ6,6-22 кВ
Генераторы электростанций, мощные электродвигателиПеременное трёхфазное10-35 кВ
Анод кинескопаПостоянное7-30 кВ
Статическое электричествоПостоянное1-100 кВ
Свеча зажигания автомобиляИмпульсное10-25 кВ
Контактная сеть электрифицированной железной дороги (Россия, переменный ток)Переменное25 кВ27,5 кВ
Пробой воздуха на расстоянии 1 см10-20 кВ
Катушка РумкорфаИмпульсноедо 50 кВ
Пробой трансформаторного масла на расстоянии 1 см100-200 кВ
Воздушная линия электропередачи большой мощностиПеременное трёхфазное35 кВ, 110 кВ, 220 кВ, 330 кВ38 кВ, 120 кВ, 240 кВ, 360 кВ
Электрофорная машинаПостоянное50-500 кВ
Воздушная линия электропередачи сверхвысокого напряжения (межсистемные)Переменное трёхфазное500 кВ, 750 кВ, 1150 кВ545 кВ, 800 кВ, 1250 кВ
Трансформатор ТеслаИмпульсное высокочастотноедо нескольких МВ
Генератор Ван де ГраафаПостоянноедо 7 МВ
Грозовое облакоПостоянноеОт 2 до 10 ГВ

См. также

Ссылки

Источник: https://dic.academic.ru/dic.nsf/ruwiki/15264

формула формула мощности электрического тока

Электрический ток является физическим процессом. Если говорить упрощенно, то это упорядоченное движение заряженных частиц. Его протекание можно измерить и соответственно выразить в символьном и цифровом виде. Формула электрического тока, представляет собой выражение качественных и количественных параметров через сопротивление проводника, напряжение или разность потенциалов, а также через его силу. Так как любое перемещение чего-либо, подразумевает под собой совершение работы, то дополнительно можно вести разговор об электричестве используя формулу мощности электрического тока.

Основные понятия и формулы характеризующие электрический ток

Количественным параметром электрического тока является его сила, представляющая собой скалярную величину и выражающуюся в отношении заряда (принято обозначать буквой q) к периоду времени (t), за которое он пересекает сечение проводника. Следовательно, формула электрического тока, а если говорить правильно его сила, будет выглядеть следующим образом — I=q/t. Измеряется данный параметр в амперах. Так как скалярные величины являются действительными числами и определяются только значением, сила тока не может иметь отрицательный знак. С учетом того, что величина заряда не является постоянным параметром для разных электрических цепей, было введено понятие – плотность электрического тока (j), формула которой выглядят так – j=I/S, где S – площадь, пересекаемая зарядами. Следовательно, при увеличении силы тока и уменьшении поперечного сечения проводника плотность тока возрастает и наоборот. Как отмечалось выше, важными параметрами электричества, вернее электрической цепи являются напряжение в ней и сопротивление проводящих ток элементов.

Формула выражения силы электрического тока через сопротивление и напряжение

В отличие от фундаментальных исследований, в основе которых лежат теоретические выкладки данная зависимость была выведена практическим путем. Автором открытия является физик Ом, в честь которого закон и получил свое имя. По результатам своих опытов и экспериментов Ом пришел к выводу что сила тока (I) напрямую зависит от величины напряжения (U)и имеет обратную зависимость от сопротивления (R) элементов и деталей, включенных в электрическую цепь. Эту связь можно представить в виде – I=U/R. Путем несложных преобразований, формулы сопротивления и напряжения, выраженные через силу тока, будут выглядеть следующим образом – R=U/I и U=IxR, соответственно.


Формула силы электрического тока

Сопротивление электрического тока: формула
Формула напряжения электрического тока

Работа и мощность электрического тока

Формула мощности (Р) электрического тока напрямую зависит от его работы (А). Под работой тока подразумевается преобразование электрической энергии в механический, тепловой, световой или иной ее вид. Величина данного процесса напрямую зависит от времени его протекания, силы тока и напряжения в сети. Это можно выразить следующей формулой – А=IxUxt. Произведение (IxU) является ничем иным как мощностью. Следовательно, чем выше напряжение или сила тока в сети, тем большую мощность имеет электрический ток и большую работу он может совершить за единицу времени. Формула мощности электрического тока имеет следующий вид – Р=А/t или Р=IxU.


Работа электрического тока формула
Формула мощности электрического тока

Поэтому, если необходимо вычислить, какую работу производит ток, протекая по цепи в течение определенного времени, необходимо умножить мощность на временной промежуток, выраженный в секундах. Рассмотрим применение формул расчета работы и мощности электрического тока на примере электрического двигателя, подключенного к сети 220 В, а сила тока, измеренная амперметром для этого участка, составила 10А.

Р (мощность двигателя) = 10А (сила тока) х 220В (напряжение в сети) = 2200 Вт = 2,2 кВт.

Зная данный показатель, а также реальное или предполагаемое время функционирования электродвигателя можно определить какую работу он совершит за этот отрезок времени или другим словами сколько будет потрачено электроэнергии. Если двигатель был включен, например, 1 час, то можно найти искомое значение.

А (работа, совершенная двигателем) = 2,2 кВт (мощность) х 1 (время работы в часах) = 2,2 кВт ч. Именно этот показатель будет отражен на приборе учета расхода электроэнергии.

Исходя из того, что электрический ток является физическим процессом, то какой-либо его неизвестный параметр можно определить, зная его остальные характеристики. Приведем наиболее распространенные формулы для определения характеристик электрической цепи применяемые в электротехнике.

Напряжение или разность потенциалов
  • U = RxI
  • U = P/I
  • U = (P*R)1/2
Сила электрического тока
Сопротивление
  • R = U / I
  • R = U2/ P
  • R = P / I2
Мощность

В заключение отметим, что приведенная информация справедлива для цепей с постоянным электрическим током. Формулы, применяемые для расчета характеристик переменного тока, будут отличаться за счет введения дополнительных переменных и характеристик свойственных данному типу электричества.

Как рассчитать выходное напряжение

Обновлено 8 декабря 2020 г.

Пол Меслер

Закон Ома – важная математическая формула, которую электрики и физики используют для определения определенных измерений в данной цепи. Формула:

В = I \ раз R

, где V – напряжение, измеренное в вольтах, I – величина тока, измеренная в амперах, а R – сопротивление, измеренное в омах. Резисторы препятствуют прохождению потока электронов в цепи и, в зависимости от их материала, обладают большим сопротивлением, чем другие.Напряжение в цепи – это не что иное, как «источник электрического потенциала» внутри этой цепи.

Цепь в серии

    Определите общую силу тока в цепи. Если у вас была цепь, и вы обнаружили, что она пропускает общий ток 6 ампер, вы должны использовать это значение в качестве силы тока в цепи. Помните, что в цепи общая сила тока везде одинакова.

    Определите общее количество сопротивлений в цепи. Вы измеряете сопротивление в омах, которое выражается греческой буквой омега.Если вы измеряете, что в этой цепи есть резистор с сопротивлением 3 Ом, а другой – с сопротивлением 2 Ом, это означает, что общее сопротивление цепи составляет 5 Ом.

    Найдите выходное напряжение, умножив силу тока на общее количество сопротивлений в цепи. В приведенных выше примерах мы знаем, что сила тока составляет 6 ампер, а общее сопротивление – 5 Ом. Следовательно, выходное напряжение для этой схемы:

    В = I \ times R = 6 \ times 5 = 30 \ text {volts}

Параллельные цепи

    Определите общий ток в цепи.Как и в последовательной цепи, ток или сила тока везде одинаковы. Используя тот же пример, мы скажем, что общая сила тока составляет 6 ампер.

    Найдите полное сопротивление в цепи. Общее сопротивление в параллельной цепи отличается от последовательной цепи. В последовательной цепи мы получаем общее сопротивление, просто добавляя каждое отдельное сопротивление в цепи; однако в параллельной цепи нам нужно найти полное сопротивление по формуле:

    R_ {tot} = \ frac {1} {\ frac {1} {R_1} + \ frac {1} {R_2} + … + \ frac {1} {R_n}}

    То есть единица, деленная на сумму обратных величин всех резисторов в параллельной цепи. Используя тот же пример, мы скажем, что резисторы имеют сопротивление 2 Ом и 3 Ом. Следовательно, полное сопротивление в этой параллельной серии составляет:

    R_ {tot} = \ frac {1} {\ frac {1} {2} + \ frac {1} {3}} = 1,2 \ text {ohms}

    Найдите напряжение так же, как вы нашли напряжение в последовательной цепи. Мы знаем, что общая сила тока для цепи составляет 6 ампер, а полное сопротивление – 1.2 Ом. Таким образом, общее выходное напряжение для этой параллельной цепи составляет:

    В = I \ times R = 6 \ times 1.2 = 7.2 \ text {volts}

Формула правила делителя напряжения, список и полное объяснение

Правило делителя напряжения следующее одна из самых распространенных концепций в проектировании электронных схем. Итак, сегодня мы подробно обсудили формулу делителя напряжения, откуда она взята, а также некоторые практические примеры. Мы также объяснили, как спроектировать схему делителя напряжения для требуемого выхода.

Изучая основы электроники, мы сталкиваемся с множеством проблем, изучая формулы, правила и шаги по их реализации. Приведенные ниже темы охватывают простой метод изучения формул, а также приемы их запоминания.

Что такое правило делителя напряжения?

Правило делителя напряжения также называется правилом делителя потенциала, правилом деления потенциала или правилом деления напряжения.
Короче он назначен как VDR. Правила для делителей напряжения
дают представление о принципиальной схеме, применимой формуле и ее выводе, чтобы помочь с различными требованиями к напряжению при проектировании схемы.

Определение делителя напряжения:

Он определяется как схема, которая используется для уменьшения большого значения напряжения до меньшего.

Он дает необходимое выходное напряжение как долю входного напряжения, которой можно управлять с помощью формулы.

Схема делителя напряжения – это схема, которая делит одно значение напряжения на несколько выходных значений.

Тип схемы:

Пассивный по своей природе (так как не имеет активных элементов)
Линейное поведение (выход линейно пропорционален входу)

Схема делителя напряжения:

Рис (а), Рис ( б) и рис (в) представляют собой принципиальные схемы делителя напряжения.Почему три схемы ниже для одного и того же правила?
Итак, ответ, это всего лишь одна схема с разным расположением и символом источника. Просто упростив их, вы обнаружите, что они одинаковы в электрических соединениях.

Анализ и формула правила делителя напряжения:

Рисунок, показывающий базовую схему цепи делителя напряжения с двумя резисторами:

Это основная принципиальная схема, которая показывает VDR и его формулу. Это очень прикладная схема, и формула обычно используется для расчета выходного напряжения повсюду при анализе цепей

Вывод делителя напряжения:

Здесь напряжение питания составляет В, подключается последовательно с резистором r1 и r2 .

И ток « протекает через них, вызывая падение напряжения v1 на r1 и падение напряжения v2 на r2 .
Поскольку это замкнутый контур, текущий ток будет таким же.
Для получения формулы выходного напряжения нам необходимо применить закон Ома к каждому резистору и поместить значения в уравнение, полученное с помощью KCL (закон Куррента Кирхгофа), как показано ниже, шаг за шагом:

Согласно закону Ома мы получаем
v1 = i ☓r1 ———- (I)
v2 = i☓r2 ———– (II)

Применение KVL в приведенной выше схеме
V – v1 – v2 = 0
т.е. V = v1 + v2
Положив значения v1 и v2 в приведенном выше уравнении,
получаем,
V = i☓r1 + i☓r2
∴ V = i☓ (r1 + r2)

Следовательно,
i = V / (r1 + r2)

Подставляя значение «i» в (I) и (II)
получаем,
v1 = r1☓ (V / (r1 + r2))
v2 = r2☓ (V / (r1 + r2))

(регулируя переменных)
Также
v1 = V☓ (r1 / (r1 + r2))
v2 = V☓ (r2 / (r1 + r2)) → (примечание: v2 = Vout) → (III)

(путем настройки переменных для условий, где нам нужно найти номиналы резисторов)
Опять же,
r1 = (v1☓ (r1 + r2)) / V
r2 = (v2☓ (r1 + r2)) / V

9 0002 [примечание: приведенная выше формула очень важна и полезна при проектировании схемы делителя напряжения]

Вывод по схеме делителя напряжения:

  • Из уравнения → (III) можно сказать, что выход напряжение равно , падение напряжения на выходном резисторе (резистор, через который мы принимаем выходной сигнал)
    (проверьте схему с 3 последовательно включенными резисторами, вы получите точку)
  • Значения резистора в знаменателе не что иное, как эквивалент резистора r1 и r2, это может быть r1 + r2 + r3 +… + rn, где n – количество резисторов.

Рисунок, показывающий делитель напряжения с 3 резисторами и его эквиваленты:

В этой схеме (согласно приведенному выше выводу из выводов):

→ Как на рис. 1 Vout1 – это напряжение на резисторе R2 и R3

∴ взяты эквивалентные последовательные сопротивления R2 и R3 .

то же самое для рис.1 (а)

→ На рис.1 Vout2 – напряжение только на резисторе R3
взято эквивалентное последовательное сопротивление R3 .

То же, что и на рис. 1 (b)

Практический пример схемы делителя напряжения (VDR) / FAQ:

Разработайте делитель напряжения, чтобы получить выходное напряжение 1,5 В для разработки усилителя смещение. Заданное напряжение источника 5В.

Дано → Vo = 1,5 В и Vin = 5 В
из уравнения → (III) или по упрощенной формуле (ищите 1-е изображение сообщения)

у нас есть, Vo = Vin. (R2 / (R1 + R2))

Допустим, R1 = 1 кОм
поместите все значения в формулу : 1.5 = 5. (R2 / (1K + R2))
Получаем, R2 = 0,428 кОм

Теперь спроектируйте схему, как показано выше !!!

Разработайте делитель напряжения, чтобы выдавать различное выходное напряжение 3 вольт и 6 вольт для компаратора, учитывая, что источник входного напряжения имеет разность потенциалов 9 вольт.

Как одинаковый последовательно включенный резистор обеспечивает одинаковое падение напряжения на каждом резисторе.
∴ в соответствии с вопросом,

Vin = 9 Вольт, Vout1 = 6Volts и Vout2 = 3Volts

Из этого мы можем сделать вывод, что наименьший выход составляет 3 вольта, а другое необходимое выходное напряжение – 6 вольт.
Затем мы можем использовать три резистора с одинаковыми номиналами. (Скажем, 1 кОм )
∴ R1 = R2 = R3 = 1 кОм Разработка завершена.

Разработайте делитель напряжения так, чтобы выходное напряжение равнялось половине входного. Заданное напряжение источника – 12 В.

Дано → Vo = 1 / 2Vin и Vin = 12 В

с использованием упрощенной формулы:
мы имеем, Vo = Vin. (R2 / (R1 + R2))

Предположим, R1 = 10 кОм
положить все значения в формуле ∴ 6 = 12. (R2 / (10K + R2))
Получаем, R2 = 10KΩ

Теперь спроектируйте схему с этими компонентами !!

Можно ли применить правило делителя напряжения в параллельных цепях?

Нет, не может применить деление напряжения по правилу в параллельной цепи , поскольку это применимо только к резисторам, включенным последовательно.Только причина того, что VDR – это модификация закона Ома.

Применяется ли правило делителя напряжения только к резисторам?

Нет, его можно применить к любому пассивному элементу, например, конденсатору и катушке индуктивности. Единственное, что вы должны предположить, это их импеданс (Z).
Вместо резистора в правиле делителя напряжения необходимо использовать импеданс вместе с модифицированной формулой уравнений импеданса.
Zr для резистора, Zc для конденсатора, Zl для индуктора.

Применение правила / схемы делителя напряжения:

1) Он используется в качестве цепи смещения в усилителе BJT.

2) Схема обратной связи в операционном усилителе использует правило делителя напряжения для управления входом и управления усилением напряжения.

3) Это важная схема компаратора, которая используется для сравнения различных напряжений, независимо от того, больше или меньше конкретное напряжение, чем опорное напряжение.

4) Сдвиг логического уровня использует формулу делителя напряжения.

Бонусные подсказки:

  • Когда резисторы R1 и R2 одинаковые, т.е. того же значения, то выходное напряжение составляет ровно половину (50%) исходного входного.
  • Кроме того, эта схема подразумевает, что она полезна там, где у нас нет источника более низкого напряжения.
  • Его можно использовать в качестве замены трансформатора (только и только если нагрузка имеет большой резистивный импеданс, мы должны использовать резисторы высокой мощности, скажем 3 Вт / 5 Вт, он практически не используется, поскольку он обладает риском поражения электрическим током. )
  • Вы заметили тестер электрических линий (тестер проводов под напряжением), это не что иное, как делитель напряжения с резистором R1 и резистором R2, который заменен световым индикатором с низким сопротивлением, потребляющим ток.

Из приведенного выше содержания мы узнали:
Что такое правило делителя напряжения?
Делитель напряжения на 3 резистора.
Вывод правила делителя напряжения.
Пример решения делителя напряжения / решенные проблемы.

Общие формулы для электроники и электричества


Ниже приведена справочная таблица, в которой приведены все уравнения, вытекающие из закона Ома. В параметры E, I, R и P показаны в центральной области, каждый из которых занимает одно из четырех квадранты пирога.Чтобы найти данный параметр, найдите этот параметр в центре диаграммы и выберите уравнение в его квадранте, которое определяет количество в терминах что вы измерили или знаете.

ПРИМЕЧАНИЕ: Уравнения, относящиеся к мощности и импедансу, описывают Коэффициент мощности (PF), а не чистый постоянный ток. Эта величина составляет реактивное сопротивление нагрузка и сигнал переменного тока.

Закон Ом (постоянный ток):

Ток в амперах = напряжение в вольтах / сопротивление в омах = мощность в ваттах / напряжение в вольт

Ток в амперах = (мощность в ваттах / сопротивление в Ом)

Напряжение в вольтах = ток в амперах Сопротивление в омах

Напряжение в вольтах = мощность в ваттах / ток в амперах

Напряжение в вольтах = (Мощность в ваттах Сопротивление в Ом) Мощность в ваттах = (Ток в амперах) 2 Сопротивление в Ом

Мощность в ваттах = Напряжение в вольтах Ток в амперах

Мощность в ваттах = (Напряжение в вольтах) 2 / Сопротивление в Ом

Сопротивление в омах = напряжение в вольтах / ток в амперах

Сопротивление в омах = мощность в ваттах / (ток в амперах) 2

Закон

Ом (переменный ток):

В следующих формулах закона Ома переменного тока q – фазовый угол в степени отставания тока от напряжения (в индуктивной цепи) или отводов тока напряжение (в емкостной цепи).В резонансном контуре (например, в обычном домашнем 120 В переменного тока) фазовый угол равен 0, а полное сопротивление = сопротивление.

Ток в амперах = напряжение в вольтах / полное сопротивление в омах

Ток в амперах = (мощность в ваттах / полное сопротивление в Ом cosq)

Ток в амперах = мощность в ваттах / (напряжение в вольтах, cos q)

Напряжение в вольтах = ток в амперах Импеданс в омах

Напряжение в вольтах = мощность в ваттах / (ток в амперах cos q)

Напряжение в вольтах = ([Мощность в ваттах Полное сопротивление в Ом] / cos q)

Импеданс в Ом = Напряжение в вольтах / Ток в амперах

Импеданс в Ом = Мощность в ваттах / (Ток в амперах 2 cos q)

Импеданс в Ом = (Напряжение в вольтах 2 cos q) / Мощность в ваттах

Мощность в ваттах = ток в амперах 2 Импеданс в омах cos q

Мощность в ваттах = ток в амперах Напряжение в вольтах cos q

Мощность в ваттах = ([Напряжение в вольтах] 2 cos q ) / Импеданс в Ом

УРАВНЕНИЯ ЭЛЕКТРОННОЙ ЦЕПИ:

Резонансная частота в герцах (где X L = X C ) = 1 / (2p [Индуктивность в Генри Емкость в Фарадах])

Реактивное сопротивление в Ом. х
X L = 2p (частота в герцах, индуктивность в генри)

Реактивность в омах емкости составляет X C
X C = 1 / (2p [частота в герцах Емкость в фарадах])

Импеданс в Ом (серия) = (Сопротивление в Ом 2 + (X L -X C ) 2 )

Импеданс в Ом (параллельно) = (Сопротивление в Ом Реактивное сопротивление) / (Сопротивление в Ом 2 + Реактивное сопротивление 2

Последовательные резисторы (значения в Ом):

Общее сопротивление = Сопротивление 1 + Сопротивление 2 +… Сопротивление n

Два параллельных резистора (значения в Ом):

Общее сопротивление = Сопротивление 1 Сопротивление 2 / Сопротивление 1 + Сопротивление 2

Параллельное подключение нескольких резисторов (значения в Ом):

Общее сопротивление = 1 / (1 / Сопротивление 1 ] + 1 / Сопротивление 2 + … 1 / Сопротивление n ])

Параллельные конденсаторы (значения в микрофарадах):

Общая емкость параллельно (значения в любых фарадах) = Емкость 1 + Емкость 2 +…. Емкость n

Последовательные конденсаторы (значения в микрофарадах):

Общая емкость в серии (значения в любых фарадах) = Емкость 1 Емкость 2 / Емкость 1 + Емкость 2

Несколько последовательных конденсаторов (значения в фарадах) = 1 / ([1 / Емкость 1 ] + [1 / Емкость 2 ] + …… [1 / Емкость n ])

Цепи времени серии LCR:
Время в секундах = индуктивность в генри / сопротивление в омах
Время в секундах = емкость в фарадах Сопротивление в омах

НАПРЯЖЕНИЕ И ТОК СИНУСОВОЙ ВОЛНЫ:

Эффективное (среднеквадратичное) значение = 0.707 Пиковое значение

Эффективное (RMS) значение = 1,11 Среднее значение

Среднее значение = 0,637 Пиковое значение

Среднее значение = 0,9 Эффективное (RMS) значение

Пиковое значение = 1,414 Эффективное (RMS) значение

Пиковое значение = 1,57 Среднее значение

DECIBELS:

дБ = 10 Log 10 (мощность в ваттах # 1 / мощность в ваттах # 2)

дБ = 10 log 10 (коэффициент мощности)

дБ = 20 log 10 (Вольт или Ампер # 1 / Вольт или Ампер # 2)

дБ = 20 Log 10 (Напряжение или коэффициент тока)

Коэффициент мощности = 10 (дБ / 10)

Коэффициент напряжения или тока = 10 (дБ / 20) Если импедансы не равны: дБ = 20 Лог 10 [(Вольт 1 [Z 2 ]) / (Вольт 2 [Z 1 ])]

Частота и длина волны

Частота в килогерцах = (300000) / длина волны метров

Частота в мегагерцах = (300) / длина волны в метрах

Частота в мегагерцах = (964) / длина волны в футах

Длина волны в метрах = (300000) / частота в килогерцах

Длина волны в метрах = (300) / частота в мегагерцах

Длина волны в футах = (964) / частота в мегагерцах

Длина волны = скорость звука (фут / сек или м / сек) / частота Скорость звука = 1130 фут / сек

Длина антенны:

Четвертьволновая антенна: (обычный провод, коэффициент скорости = 0.95)
Длина в футах = 234 / частота в мегагерцах

Полуволновая антенна: (обычный провод, коэффициент скорости = 0,95)
Длина в футах = 466 / частота в мегагерцах

Сопротивление первичной обмотки согласующего трансформатора громкоговорителя на 70 В = (Выход усилителя вольт) 2 / Speaker Power

Electrical Voltage – Web Formulas

Электрическое напряжение также называется электрическим потенциалом и определяется как разность электрических потенциалов между двумя электрическими полюсами батареи во время протекания электрического тока.

Рассмотрим на примере аккумуляторной батареи; Аккумулятор состоит из двух электрических полюсов – положительного и отрицательного электрического полюса. Химический процесс, происходящий в батарее, приводит к химической или неэлектрической силе. Благодаря химической силе положительные заряды движутся к положительным полюсам, а отрицательные заряды движутся к отрицательному полюсу и накапливаются там. В процессе накопления зарядов на соответствующих полюсах возникает разность электрических потенциалов между двумя полюсами, которая постепенно увеличивается.Как только разность электрических потенциалов достигает максимума и больше не происходит накопления зарядов на соответствующих полюсах, в это время разность электрических потенциалов и химическая сила становятся равными. Разность электрических потенциалов в этой точке называется ЭДС – электродвижущая сила батареи.

ЭДС единица измерения: джоуль / кулон = вольт. Название дано в памяти ученого Вольта.

Предположим, мы присоединяем аккумулятор с помощью проводящего провода, после чего создается электрическое поле.Из-за электрического поля положительные заряды текут навстречу отрицательной силе и образуют электрический ток. Следовательно, разность потенциалов между двумя электрическими полюсами меняется при протекании электрического тока. Эта разность электрических потенциалов между двумя полюсами батареи во время протекания тока называется напряжением на клеммах, или электрическим напряжением, или электрическим потенциалом.

Отношение между напряжением и ЭДС определяется как
V = E – I

× r
Где у нас
V = Напряжение
E = электродвижущая сила
I = ток
r = внутреннее сопротивление

Прибор, используемый для измерения электрического потенциала или электрического напряжения между любыми двумя точками, называется вольтметром и подключается параллельно двум рассматриваемым точкам.

Примеры расчетов

Пример-1: Внутреннее сопротивление батареи 12 В составляет 0,17 Ом, когда ток, протекающий от батареи, составляет 0,1 x 10 4 мА; рассчитать напряжение на клеммах аккумулятора.

Причина:

Здесь имеем:
E = 12 В
r = 0,17 Ом
I = 0,1

Пример-2: Почему положительные заряды не переместились от положительного полюса к отрицательному полюсу батареи до соединения с проводящим проводом?

a) Положительные заряды противоположны внутреннему сопротивлению батареи
б) Положительные заряды сталкиваются с противодействием неэлектрической силы
в) Энергия положительных зарядов обращается в ноль, как только она достигает положительного заряда
d) Ни один из вышеперечисленных

Причина: Положительные заряды сталкиваются с противодействием неэлектрической силы перед соединением батареи с проводящим проводом.

Пример-3: Прибор для измерения электрического напряжения ……………

Причина: прибор для измерения электрического напряжения – вольтметр.

Напряжение в последовательной цепи

Напряжение в последовательной цепи

Напряжение, падающее на резисторе в цепи, состоящей из одного резистора и источника напряжения, представляет собой полное напряжение в цепи и равно приложенному напряжению.Общее напряжение в последовательной цепи, состоящей из более чем одного резистора, также равно приложенному напряжению, но состоит из суммы падений напряжения отдельных резисторов. В любой последовательной схеме сумма падений напряжения на резисторе должна равняться напряжению источника. Это утверждение может быть подтверждено исследованием схемы, показанной на рисунке 3-17. В этой цепи потенциал источника (E T ) в 20 вольт падает на последовательную цепь, состоящую из двух резисторов сопротивлением 5 Ом. Общее сопротивление цепи (R T ) равно сумме двух отдельных сопротивлений или 10 Ом.Используя закон Ома, ток в цепи можно рассчитать следующим образом:

Рисунок 3-17. – Расчет отдельных падений напряжения в последовательной цепи.

Поскольку известно, что сопротивление каждого резистора составляет 5 Ом, а ток через резисторы составляет 2 ампера, падение напряжения на резисторах может быть рассчитано. Таким образом, напряжение (E 1 ) на R 1 составляет:


Осмотрев цепь, вы можете увидеть, что R 2 имеет то же омическое значение, что и R 1 , и несет такой же ток.Падение напряжения на R 2 , следовательно, также равно 10 вольт. Если сложить эти два падения по 10 вольт, общее падение составит 20 вольт, что в точности равно приложенному напряжению. Для последовательной схемы:

E T = E 1 = E 2 + E 3 =. . . E n

Пример: последовательная цепь состоит из трех резисторов, имеющих номиналы 20 Ом, 30 Ом и 50 Ом соответственно. Найдите приложенное напряжение, если ток через резистор 30 Ом равен 2 ампера.(Аббревиатура amp обычно используется для обозначения ампер.)

Для решения проблемы сначала рисуется принципиальная схема и маркируется (рис. 3-18).

Рисунок 3-18. – Решение для приложенного напряжения в последовательной цепи.


Подстановка:


ПРИМЕЧАНИЕ: При использовании закона Ома величины для уравнения ДОЛЖНЫ быть взяты из ОДНОЙ части схемы. В приведенном выше примере напряжение на R 2 было вычислено с использованием тока через R 2 и сопротивления R 2 .

Величина падения напряжения на резисторе определяется приложенным напряжением и пропорционально сопротивлению цепи. Падения напряжения, возникающие в последовательной цепи, прямо пропорциональны сопротивлениям. Это результат того, что через каждый резистор протекает одинаковый ток – чем больше сопротивление резистора, тем больше падение напряжения на нем.

Последовательная цепь, состоящая из трех резисторов, имеет ток 3 ампера. Если R 1 = 20 Ом, R 2 = 60 Ом и R 3 = 80 Ом, каковы (а) полное сопротивление и (б) напряжение источника цепи?

Какое напряжение падает на каждом резисторе цепи, описанной в вопросе 17?

Если бы ток был увеличен до 4 ампер, каким было бы падение напряжения на каждом резисторе в цепи, описанной в вопросе 17?

Что нужно сделать со схемой, описанной в вопросе 17, чтобы увеличить ток до 4 ампер?

Напряжение холостого хода | PVEducation

Напряжение холостого хода, V OC , является максимальным напряжением, доступным от солнечного элемента, и это происходит при нулевом токе.Напряжение холостого хода соответствует величине прямого смещения солнечного элемента из-за смещения перехода солнечного элемента с током, генерируемым светом. Напряжение холостого хода показано на ВАХ ниже.

ВАХ солнечного элемента, показывающая напряжение холостого хода.

Уравнение для V oc можно найти, установив чистый ток равным нулю в уравнении солнечного элемента, чтобы получить:

$$ V_ {OC} = \ frac {n k T} {q} \ ln \ left (\ frac {I_ {L}} {I_ {0}} + 1 \ right) $$

Случайный просмотр приведенного выше уравнения может показать, что V OC растет линейно с температурой.Однако это не так, поскольку I 0 быстро увеличивается с температурой, в первую очередь из-за изменений собственной концентрации носителей n i . Влияние температуры сложное и зависит от технологии ячеек. См. Более подробную информацию на странице «Влияние температуры».

V OC уменьшается с температурой. Если температура изменится, I 0 также изменится.

Приведенное выше уравнение показывает, что V oc зависит от тока насыщения солнечного элемента и тока, генерируемого светом.Хотя I sc обычно имеет небольшое изменение, ключевым эффектом является ток насыщения, поскольку он может варьироваться на порядки. Ток насыщения I 0 зависит от рекомбинации в солнечном элементе. Тогда напряжение холостого хода является мерой рекомбинации в устройстве. Кремниевые солнечные элементы на высококачественном монокристаллическом материале имеют напряжение холостого хода до 764 мВ при одном солнце и условиях AM1.5, в то время как коммерческие устройства на мультикристаллическом кремнии обычно имеют напряжение холостого хода около 600 мВ.{2}} \ right] $$

, где kT / q – тепловое напряжение, N A – концентрация легирования, Δn – концентрация избыточных носителей заряда, а n i – собственная концентрация носителей. Определение V OC по концентрации носителей также называется подразумеваемым V OC .

Voc как функция ширины запрещенной зоны, E

G

Если ток короткого замыкания (I SC ) уменьшается с увеличением ширины запрещенной зоны, напряжение холостого хода увеличивается с увеличением ширины запрещенной зоны.{x} -1} d x $$,

где q – заряд электрона, σ – постоянная Стефана – Больцмана, k – постоянная Больцмана, T – температура и

$$ u = \ frac {E_ {G}} {k T} $$

Вычисление интеграла в приведенном выше уравнении довольно сложно. На приведенном ниже графике используется метод, описанный в

.

Зависимость тока насыщения диода от ширины запрещенной зоны. Значения определяются на основе подробного баланса и устанавливают ограничение на напряжение холостого хода солнечного элемента.

Вычисленное выше значение J 0 можно напрямую подключить к стандартному уравнению солнечного элемента, приведенному в верхней части страницы, для определения V OC , если напряжение меньше ширины запрещенной зоны, как в случае с солнечное освещение.

V OC как функция ширины запрещенной зоны для соты с AM 0 и AM 1.5. V OC увеличивается с шириной запрещенной зоны по мере уменьшения рекомбинационного тока. В V OC наблюдается спад при очень большой ширине запрещенной зоны из-за очень низкого значения I SC .

Расчет кода

| EC&M

В прошлом месяце мы рассмотрели, как NEC решает проблему падения напряжения, обсудив шесть примечаний к мелкому оттиску (FPN) и несколько конкретных правил. В этом месяце мы рассмотрим примеры, демонстрирующие, как рассчитать падение напряжения на проводниках, используя закон Ома или формулу падения напряжения (VD).

Метод закона Ома: только однофазный. Падение напряжения в проводниках цепи можно определить, умножив силу тока цепи на общее сопротивление проводов цепи: VD = I x R.«I» равно нагрузке в амперах, а «R» равно сопротивлению проводника, как указано в таблице 8 главы 9 для цепей постоянного тока или в таблице 9 главы 9 для цепей переменного тока. Примечание. Вы не можете использовать метод закона Ома для трехфазных цепей.

Однофазный, 120 В, пример: Какое падение напряжения на двух проводниках № 12, питающих нагрузку 16 А, 120 В, расположенных на расстоянии 100 футов от источника питания (200 футов провода)?

(а) 3,2 В
(б) 6,4 В
(в) 9.6В
(г) 12,8В

Ответ: (б) 6.4V

Падение напряжения = I x R, где «I» равно 16 А и

«R» равно 0,4 Ом [Глава 9, Таблица 9: (2 Ом / 1000 футов) 2200 футов 42200 футов]

Следовательно, падение напряжения = 16A x 0,4 Ом или

Падение напряжения = 6,4 В, (6,4 В / 120 В = падение напряжения 5,3%) и

Рабочее напряжение = 120 В – 6,4 В или

Рабочее напряжение = 113,6 В

В.Д. Формульный метод. Если вы уже установили проводники цепи, определите падение напряжения на проводниках по одной из следующих формул:

VD = 2 x K x Q x I x D / CM (однофазный) или
VD = 1.732 x K x Q x I x D / CM (3-фазный), где «VD» = падение напряжения: мы выражаем падение напряжения на проводниках цепи в вольтах.

«K» = Постоянная постоянного тока: это постоянная, представляющая сопротивление постоянному току для проводника круглого сечения 1000 мил и длиной 1000 футов при рабочей температуре 75 ° C. Постоянное значение постоянного тока, которое вы должны использовать для меди, составляет 12,9 Ом и 21,2 Ом для алюминиевых проводов. Константа «K» подходит для цепей переменного тока, где проводники не превышают №1/0.

«Q» = Поправочный коэффициент переменного тока: Вы должны отрегулировать цепи переменного тока № 2/0 и выше на эффекты самоиндукции (скин-эффект). Коэффициент регулировки «Q» можно найти, разделив сопротивление переменному току, как указано в главе 9, таблица 9, на сопротивление постоянному току, как указано в главе 9, таблица 8.

«I» = Амперы: нагрузка в амперах при 100%, а не 125% для двигателей или постоянных нагрузок.

«D» = Расстояние: расстояние, на котором находится нагрузка от источника питания, а не общая длина проводников цепи.

«CM» = Круглые милы: Круговые милы проводника цепи, как указано в главе 9, таблица 8.

Однофазный пример: Какое падение напряжения для проводника № 6, который питает однофазную нагрузку 44 А, 240 В, расположенную на расстоянии 160 футов от щитка?

(а) 4,25 В
(б) 6,9 В
(в) 3%
(г) 5%

Ответ: (б) 6.9В

VD = 2 x K x I x D / CM, где
K = 12,9 Ом (медь),
I = 44 A,
D = 160 футов и
CM = 26 240 круглых милов (Глава 9, Таблица 8)

Следовательно, VD = 2 провода х 12.9 Ом x 44A x 160 футов / 26240 круглых мил, или
В = 46,9 В, (6,9 В / 240 В = падение напряжения 2,9%), и
Рабочее напряжение = 240 В – 6,9 В или
Рабочее напряжение = 233,1 В

Пример трехфазной сети: Предположим, у вас есть трехфазная нагрузка 208 В, 36 кВА (100 А), расположенная в 80 футах от щитка и соединенная алюминиевыми проводниками № 1. Какое падение напряжения в проводниках до отключения оборудования?

(а) 3,5 В
(б) 7 В
(в) 3%
(г) 5%

Ответ: (а) 3.5В

VD = 1,732 x K x I x D / CM, где
K = 21,2 Ом (алюминий),
I = 100A,
D = 80 футов и
CM = 83690 круглых милов (глава 9, таблица 8)

Следовательно, VD = 1,732 x 21,2 Ом x 100A x 80 футов / 83690 круглых мил или
VD = 3,5 В (3,5 В / 208 В = 1,7%), а рабочее напряжение
= 208–3,5 В или
Рабочее напряжение = 204,5 В.

Добавить комментарий

Ваш адрес email не будет опубликован. Обязательные поля помечены *